Emergency PA Easy

¡Supera tus tareas y exámenes ahora con Quizwiz!

55-year-old woman with a history of mitral stenosis, secondary to rheumatic heart disease, presents to the emergency department with increasing dyspnea while walking up one flight of stairs. She denies chest pain and discomfort, but states that recently she has also noticed palpitations. She also admits to lower extremity edema, which is new within the last week. Which of the following tachyarrhythmias is she most likely to demonstrate on EKG? A Ventricular tachycardia B Atrial flutter C Ventricular fibrillation D Ventricular bigeminy E Torsades de pointes

B Choice B is correct. As patients with mitral stenosis age, and their mitral stenosis progresses to moderate or moderately severe mitral stenosis (most commonly after their fourth decade), the incidence of atrial arrhythmias—including premature atrial contractions, paroxysmal tachycardia, atrial flutter, and atrial fibrillation—increases.

A 76-year-old man with a history of HTN and diabetes mellitus, type 2, presents to the emergency department with complaints of palpitations, tachypnea, and chest pain. He denies a history of CAD, stroke, TIA, or congestive heart failure. He is afebrile, with vital signs as follows: BP 145/98, HR 138, and RR 22. His EKG is shown (Figure 1). Troponins are negative X 1. His echocardiogram demonstrates normal LV systolic function and normal valvular function. Based on his electrocardiogram, which of the following is his most likely diagnosis? A Normal sinus rhythm at 65 bpm with premature atrial complexes B Atrial fibrillation with a rapid ventricular rate of approximately 140 bpm C Atrial flutter with 2:1 conduction D Sinus tachycardia at a rate of approximately 120 bpm E Ventricular fibrillation

B Choice B is correct. This patient's EKG demonstrates atrial fibrillation, with disorganized atrial activity, as evidenced by the lack of discrete p waves and irregularly irregular ventricular activity. Choice A, normal sinus rhythm with PACs, would demonstrate a somewhat irregular rhythm, but discrete p waves would be noted. Choice C, atrial flutter, would be characterized by a regular ventricular rate with a sawtooth morphology. Choice D, sinus tachycardia, would demonstrate a regular, though fast, ventricular rate with discrete p waves. Choice E, ventricular fibrillation, would be an irregularly irregular rhythm; however, QRS complexes would not be clearly defined.

A 45-year-old male who presents to the emergency department with sudden onset of lip swelling, which began shortly after awakening this morning. He denies any history of allergies and denies any new medications. His current medications include hydrochlorothiazide (HCTZ), captopril, atenolol, atorvastatin, and fexofenadine. What is the most likely cause of this gentleman's symptoms? Source: (Knoop et al., 2010, Chapter 5) A HCTZ B captopril C atenolol D atorvastatin E fexofenadine

B The correct answer is (B). ACE inhibitor angioedema is a potentially life-threatening known side effect of captopril. The patient may need emergency intubation to ensure that his airway remains patent. Patients with a history of ACE inhibitor angioedema should not be placed on any ACE inhibitor or an ARB due to the possibility of similar reaction. HCTZ (a diuretic) and atenolol (a beta blocker) are antihypertensives that are generally unlikely to be associated with angioedema. Atorvastatin (a statin) and Fexofenadine (an antihistamine) are unlikely to present with angioedema.

A 66-year-old female with a history of nephrotic syndrome presents to the emergency department complaining of a non-productive cough and dyspnea on exertion following a recent vacation to Orlando. She is currently taking prednisone. What diagnosis is highly suspected in this patient due to her history of nephrotic syndrome? A pneumonia B pulmonary embolism C chronic obstructive pulmonary disease (COPD) exacerbation D Cushing's syndrome E asthma exacerbation

B The correct answer is (B). Patient's with nephrotic syndrome commonly have a hypercoagulable state and are at risk for deep venous thrombosis (DVT) with resultant PE. Patients on chronic prednisone may have a risk of developing Cushing's syndrome, but this is not consistent with the patient's history. Pneumonia is possible due to chronic prednisone use but not directly caused by her history of nephrotic syndrome. COPD and asthma are not linked to nephrotic syndrome.

A teenage girl presents to the emergency department with her parents. She has had symptoms of a urinary tract infection for the last two days, but did not tell her parents until today. She is not sexually active. Today, the girl also has diffuse abdominal pain with vomiting, general malaise, and difficulty breathing. She has no significant past medical history. Her physical exam reveals sinus tachycardia and deep fast respirations with no localization of abdominal pain or rebound tenderness. Initial lab test results reveal a plasma glucose = 378 mg/dL and serum bicarbonate = 14 mEq/L. What is the most likely diagnosis? A Hyperosmolar nonketotic hyperglycemia B Diabetic ketoacidosis C Chronic corticosteroid use D Gestational diabetes E Schmidt syndrome

B The correct choice is B, diabetic ketoacidosis (DKA). Classic signs and symptoms of this disorder include polyuria, polydipsia, marked fatigue, nausea, vomiting, signs of dehydration, fruity breath odor, postural hypotension, Kussmaul respirations, and possibly mental stupor or coma. Patients with type 1 diabetes mellitus may present for the first time in DKA. DKA is commonly precipitated by a recent infection. Choice A, hyperosmolar nonketotic hyperglycemia, is seen in patients with type 2 diabetes and presents with extremely high plasma glucose levels without acidosis and ketosis. Choice C, chronic corticosteroid use, can cause hyperglycemia and possible glucose intolerance or diabetes mellitus. This patient has no history of oral corticosteroid use. Choice D, gestational diabetes, occurs in women who are pregnant. Choice E, Schmidt syndrome, is an autoimmune polyglandular syndrome which includes diabetes mellitus in greater than 50% of patients.

How many days after implantation is beta-hCG detectable?

Beta-hCG is secreted from the time of implantation and is detectable about 7-8 days after fertilization.

A 76-year-old man with a history of HTN and diabetes mellitus, type 2, presents to the emergency department with complaints of palpitations, tachypnea, and chest pain. He denies history of CAD, stroke, TIA, or congestive heart failure. He is afebrile, with vital signs as follows: BP 145/98, HR 138, and RR 22. His EKG is shown (Figure 1). Troponins are negative X 1. His echocardiogram demonstrates normal LV systolic function and normal valvular function. He states that his symptoms began 3 days ago, and had gotten worse in the last 2 hours. What would be the most appropriate next step in management? A Morphine, oxygen, nitroglycerin sublingually, aspirin 81 mg PO X 4 B Reassurance and anxiolytics C Diltiazem 20 mg bolus IVP over 2 minutes, then diltiazem 10 mg/h IV infusion D Aspirin 81 mg two tablets PO E Dopamine 5 mcg/kg/min

C Choice C is correct; as the patient is demonstrating atrial fibrillation with RVR, the appropriate therapy now that acute myocardial infarction has been ruled out, and as the patient is beyond the therapeutic window for immediate direct current electrocardioversion, is heart rate control. Choice A is inappropriate, as this is standard therapy for acute MI. Choice B can be used in patients suffering palpitations associated with panic attacks. Choice D is insufficient anticoagulation therapy. Choice E is inappropriate as the patient is not in need of pressor support

A 56-year-old male, with history of hyperlipidemia and non-insulin-dependent diabetes mellitus (NIDDM) presents to the emergency department with a history of increasing peripheral edema over the past week. On examination he is noted to have periorbital, scrotal, and +2 pretibial edema. His lungs are CTAB. He denies any chest pain or shortness of breath. Urine dipstick reveals 4+ protein. Urine microscopic reveals Maltese crosses consistent with lipiduria. Labs include a decreased serum albumin of 2 g/dl, decreased total protein of 5.5 g/dl, and normal glomerular filtration rate (GFR). What is the most likely diagnosis? A pyelonephritis B congestive heart failure (CHF) C nephrotic syndrome D prostatitis E Deep venous thrombosis (DVT)

C The correct answer is (C). This patient has typical symptoms of nephrotic syndrome, which includes significant proteinuria, hypoalbuminemia, and typical presentation of edema. He also has a history of hyperlipidemia and laboratory findings of lipiduria, which is also common in nephrotic syndrome. Furthermore, his history of diabetes mellitus is also a potential cause of nephrotic syndrome. Pyelonephritis and prostatitis would present with urine WBCs and is not consistent with the laboratory findings or edema. CHF would more likely present with dyspnea, rales on exam, and peripheral edema but would unlikely involve the periorbital area. DVT would likely present with unilateral swelling of the LE, and discomfort and is not consistent with the laboratory findings above.

You are evaluating a 67-year-old male with known cirrhosis of the liver secondary to alcoholic liver disease, although he has been sober for the past year. He is brought in to the emergency department by his daughter, who notes that for the past few days he has seemed to be more confused. On examination you note the patient to be mildly confused but alert to person and place. He has noticeable asterixis. He is not currently taking any medications and his blood alcohol level is undetectable. What is the treatment of choice in this case based on your physical examination findings? A amoxicillin B prednisone C lactulose D folic acid E thiamine

C The correct answer is (C). This patient most likely has hepatic encephalopathy due to end-stage liver disease. Asterixis indicates an increase in serum ammonia. The treatment of choice is lactulose. Both folic acid and thiamine are used in the treatment of alcoholic liver disease, but do not treat elevated ammonia levels. Antibiotics may be used secondarily in patients nonresponsive to lactulose, but amoxicillin is not preferred. Prednisone is not a treatment for hepatic encephalopathy.

Which of the following is the greatest risk factor for an ectopic pregnancy? AMultiple sexual partners BPrevious abdominal surgery CPrevious ectopic pregnancy DPrevious pelvic inflammatory disease

Correct Answer ( C ) Explanation: An ectopic pregnancy is when implantation of the gestational sac occurs outside of the uterus. The most common location for an ectopic pregnancy is the fallopian tubes. It can also occur in the interstitial or cornual portion of the uterus (2%), intraabdominally (1.5%), on the ovary (0.1%) and or within the cervix (0.1%). A history of a previous ectopic pregnancy is regarded as the greatest risk factor for an ectopic pregnancy with an odds ratio of 8.3. Other high-risk features include previous tubal surgery and in utero diethylstilbestrol (DES) exposure. Ectopic Pregnancy Patient will be a woman With a history of prior ectopic, PID, tubal surgery, IUD Complaining of vaginal bleeding, abdominal pain, amenorrhea PE will show adnexal tenderness or unexplained hypotension Labs will show positive pregnancy test and lower than expected serum ß-hCG levels Diagnosis is made by ultrasound Most commonly located in a fallopian tube Treatment is MTX or surgery

50-year-old woman with a history of hypertension complains of chest tightness and dyspnea while walking up one flight of stairs. She recently experienced an episode of near-syncope. She denies a history of rheumatic fever. On auscultation, a crescendo-decrescendo systolic ejection murmur is heard at the upper right sternal border radiating to the carotids bilaterally. Given the patient's physical exam findings, which of the following is the most likely diagnosis? A Severe aortic stenosis secondary to congenital bicuspid aortic valve B Mitral regurgitation/insufficiency C Mitral stenosis D Aortic regurgitation/insufficiency E Tricuspid regurgitation/insufficiency

A Choice A is correct, as the murmur of aortic stenosis is usually described as a crescendo-decrescendo or systolic ejection murmur heard best at the right upper sternal border. In addition, the murmur of aortic stenosis is frequently transmitted to the carotid arteries. In a patient of this age, with symptoms suggestive of severe aortic stenosis and with these physical exam findings, a congenital bicuspid aortic valve is the best choice, especially as the patient denies a history or rheumatic fever, which is also a cause of developing aortic valve stenosis in individuals under the age of 65. Patients with a congenital bicuspid aortic valve typically develop symptoms once the valve leaflets have become calcified and thickened, secondary to the undue stress over many years on a structurally abnormal aortic valve. Choice B is incorrect, as the murmur of aortic regurgitation is usually described as a high-frequency decrescendo early diastolic murmur heard best at the left upper sternal border or at the right upper sternal border. Choice C is incorrect, as the murmur of mitral stenosis is described as a low-frequency rumbling diastolic murmur that is decrescendo in early diastole, but may become crescendo up to the first heart sound with moderately severe mitral stenosis and sinus rhythm. Choice E is incorrect, as the murmur of tricuspid regurgitation is described as a holosystolic descrescendo murmur.

The Correct Answer is: A This patient has asymptomatic bigeminy. In this case, the patient is otherwise healthy and has no other medical problems. Patients with this presentation require no therapy, and this is an incidental finding on her evaluation. Only monitoring of the patient is considered.

A 33-year-old female presents to the emergency department for the complaint of abdominal pain that has been present for the last month. During her workup, she has an ECG completed. She denies any shortness of breath, palpitations, orthopnea, dyspnea on exertion, syncope, weakness, or headaches. On physical exam, she is alert, awake, and in no distress. Her vital signs are as follows: temperature is 98.9, pulse is 100, respiratory rate is 18, and blood pressure is 132/90. Her HEENT is within normal limits, her neck is supple with a slightly enlarged thyroid that is non-tender without nodules, her lungs are clear, and cardiac is a regularly irregular rhythm. Based on these findings, what is the best therapy for this patient? A No therapy B Ablation therapy C Beta blockers D Digoxin E Verapamil

A 6-year-old female presents to the emergency department with left wrist pain after falling off the monkey bars at the school playground. Imaging of the left upper extremity shows the following fracture pattern: Which type of Salter-Harris Classification is observed? A Type I B Type II C Type III D Type IV E Type V

The Correct Answer is: A A Salter-Harris Type I (A) involves the entire epiphysis. Type II (B) is the entire epiphysis along with a portion of the metaphysis, Type III (C) involves a portion of the epiphysis only, Type IV (D) involves a portion of the epiphysis along with a portion of the metaphysis, and Type V (E) is a compression injury of the epiphyseal plate (nothing is "broken off").

Two days following an uneventful 4-vessel CABG, a 57-year-old man develops a sudden onset of lightheadedness and palpitations. His vital signs are stable, and physical examination demonstrates no abnormalities. Given the results of his EKG, as shown (Figure 2), which of the following is the most appropriate next step in management? A Direct-current cardioversion B Nitroglycerin patch C Digoxin 0.125 mg PO daily D Neurology consult E Meclizine 25 mg PO Q6H

The Correct Answer is: A Among the choices offered here, choice A is the most appropriate next step in management of a patient with new onset atrial flutter, as determined by EKG; it most effectively converts most patients to normal sinus rhythm.

Patients diagnosed with an auricular hematoma are at increased risk of developing which condition? A Cartilage necrosis B Cholesteatoma C Coagulopathy D Exostosis E Otomycosis

The Correct Answer is: A Auricular hematomas occupy the subperichondral space, leading to decreased or absent diffusion from the perichondrium to the cartilage and resulting in increased risk of necrosis (A). Coagulopathy (C) may predispose a patient to experiencing a hematoma. Cholesteatoma (B) may result from TM trauma, but not blunt trauma to the outer ear.

A patient is brought to the Emergency Department by ambulance. He is a 27-year-old male who is well known to the paramedic team as a heroin addict. He is arousable and does not remain alert when aroused. Which of the following physical signs would help to confirm the diagnosis of opioid intoxication? A Bradycardia B Diaphoresis C Mydriasis D Rhinorrhea E Tachypnea

The Correct Answer is: A Bradycardia (A), lowered respiratory rate, miosis and somnolence are the main effects of opiates. As with most medications, withdrawal effects are the opposite of the effects of overdose. Tachycardia, tachypnea, rhinorrhea and diaphoresis (B, D, & E) can all occur in opiate withdrawal. Mydriasis (C) is common with other substances of abuse, such as cocaine and LSD.

48-year-old man is brought to the emergency department by his sister after suffering from loss of consciousness, followed by muscle rigidity and rhythmic contractions, and then a return to a normal state. When asked about medication use, the patient states he is currently being treated with a drug for depression but cannot remember the name. He claims that he has never had a seizure or seizure-like activity prior to this event. Approximately 6 hours after the first episode, the patient suffers a second one while still in the ED. Which of the following medications is the patient most likely taking? A bupropion B duloxetine C fluoxetine D nortriptyline E phenelzine

The Correct Answer is: A Bupropion has been shown in some patients to cause seizures in a dose-dependent fashion, particularly in those with a history of head trauma or electrolyte abnormalities. Tricyclic antidepressants (eg, nortriptyline), selective serotonin reuptake inhibitors (eg, fluoxetine), serotonin-norepinephrine reuptake inhibitors (eg, duloxetine), and monoamine oxidase inhibitors (eg, phenelzine) have not been associated with seizures.

During a hospitalization for acute exacerbation of COPD, troponin levels are drawn on a 62-year-old man with a history of hypertension, hyperlipidemia, and chronic tobacco use, and found to be elevated above the 99 th percentile of normal. Which of the following choices would qualify this patient for the most recent ACC/AHA consensus guideline's definition of myocardial infarction? A Ischemic symptoms B New right bundle branch-block on EKG C J wave on EKG D Pulmonary vascular congestion on CXR E Elevated WBC count

The Correct Answer is: A Choice A is the most appropriate choice, as troponin elevation may occur in the setting of patients who do not suffer from acute coronary syndrome. Therefore, the 2007 consensus guidelines recommended that the definition of myocardial infarction be applied to those patients who not only had troponin elevation above the 99 th percentile, but also met one of the following criteria: "ischemic symptoms, new left bundle branch block (not right bundle branch-block as in choice B), new ST and T-wave changes, new Q waves, or imaging evidence of a new loss of viable myocardium or new regional wall-motion abnormality." Choice C, J wave, is characteristic of patients with hypothermia. Choice D, pulmonary vascular congestion, is frequently noted on CXR of patients with congestive heart failure. Choice E, an elevated WBC count, is indicative of an infectious process.

A 22-year-old recent immigrant from Vietnam, who is 28 weeks pregnant with her first child, presents to the emergency department with complaints of worsening dyspnea and lower extremity edema. She is unable to answer definitively whether or not she has a history of rheumatic fever. On physical examination, a possible opening snap, loud S 1 , and a very soft diastolic rumbling murmur is auscultated. When the patient is placed in the left lateral decubitus position, the murmur is accentuated, and heard best at the apex. With inspiration, the murmur does not increase in amplitude. On echocardiogram, mitral stenosis is noted. Which of the following is the most appropriate next step in management of this patient? A Beta blockade and support stockings B ACE inhibitor C Mitral valve replacement D Cardiac catheterization E Increased sodium intake to maintain fluid volume

The Correct Answer is: A Choice A is the most appropriate next step in the management of this pregnant patient with mitral stenosis. In pregnancy, blood volume, cardiac output, and heart rate are increased. In pregnant patients with mitral stenosis, this increases the pressure across the mitral valve and can lead to pulmonary edema. The use of appropriate beta blockade is helpful for decreasing the heart rate, and may be used in conjunction with digoxin if the patient develops atrial fibrillation, a common dysrhythmia in this patient population. Support stockings are helpful in preventing venous pooling in the lower extremities, which can lead to large fluctuations in hemodynamics. Choice B, ACE inhibitors, are contraindicated in pregnancy, secondary to the teratogenic effects. Choice C, mitral valve replacement, is indicated only if medical management is insufficient to prevent congestive heart failure, a serious consideration in this valvulopathy, which is the most likely to cause death in pregnancy. Choice D, cardiac catheterization, is used for evaluation of coronary artery disease, but is not indicated in this young patient with few risk factors for coronary artery stenosis. Choice E, increased sodium intake to maintain fluid volume, would worsen the pressure across the mitral valve, and thus is incorrect, as avoidance of fluid overload is one of the keys to management of a pregnant patient with mitral stenosis, especially in the latter stages of pregnancy. Patients who are well-controlled throughout pregnancy, with medical management, are still at risk during labor and delivery, when large fluctuations in hemodynamics will occur

A 24-year-old HIV-positive man comes to the emergency department complaining of severe left-sided chest discomfort, which radiates through to the left trapezius region. On coming into the room, you note that he is sitting up and hunched forward. On physical examination, the patient's blood pressure is 135/78, with a pulse of 85 bpm, and a pericardial friction rub is noted. Laboratory findings demonstrate elevated serum creatine kinase levels and normal serial troponin levels. His EKG demonstrates global ST segment elevation. His CXR demonstrates no acute process. Which of the following is the most likely diagnosis in this patient? A Acute pericarditis B Acute myocardial infarction C Acute bacterial endocarditis D Acute ascending aortic dissection E Acute costochondritis

The Correct Answer is: A Choice A is the most likely finding, as this patient is exhibiting signs, symptoms, and EKG findings pathognomonic for acute pericarditis, which is likely infectious in the setting of a patient with HIV. A pericardial friction rub is heard best with the patient in a seated position, during expiration, and is frequently found in patients with pericarditis. Choice B, an acute myocardial infarction, is less likely in a patient of this age, especially with normal serial troponins. Acute pericarditis can sometimes present with elevated serum creatine kinase levels when the epicardium is also involved. Choice C, acute bacterial endocarditis, is less likely in a patient with these EKG changes. Choice D, aortic dissection, would present with chest pain; however, the patient would be markedly hypotensive, less stable on presentation, and a CXR would demonstrate widening of the superior mediastinum

A 24-year-old HIV-positive man comes to the emergency department complaining of severe left-sided chest discomfort, which radiates through to the left trapezius region. On coming into the room, you note that he is sitting up and hunched forward. On physical examination, the patient's blood pressure is 135/78, with a pulse of 85 bpm, and a pericardial friction rub is noted. Laboratory findings demonstrate elevated serum creatine kinase levels and normal serial troponin levels. His EKG demonstrates peaked T waves. His CXR demonstrates a "water bottle" cardiac silhouette. Which of the following are serious consequences of acute pericarditis, which require careful monitoring? A Pericardial effusion B Aortic dissection C Myxedema D Chylopericardium E Acute myocardial infarction

The Correct Answer is: A Choice A, pericardial effusion, is a serious consequence of acute pericarditis, which requires careful monitoring to ensure that progression of the pericardial effusion does not lead to cardiac tamponade, which can be fatal if not treated promptly. Choice B is unlikely in a patient with acute pericarditis. Choices C, D, and E are noninfectious causes, not consequences of pericarditis.

A 66-year-old man with a history of HTN and diabetes mellitus, type 2, presents to the emergency department with complaints of palpitations for over 2 weeks, tachypnea, and chest pain. He denies history of CAD, stroke, TIA, or congestive heart failure. He is afebrile, with vital signs as follows: BP 145/98, HR 138, and RR 22. His EKG is shown (Figure 1). Troponins are negative X 3. Which of the following choices is the most appropriate next diagnostic study for this patient? A Transthoracic echocardiogram B Cardiac catheterization C Nuclear stress test D Holter monitor E Event recorder

The Correct Answer is: A Choice A, transthoracic echocardiogram, is correct, as it can demonstrate the presence of valvular heart disease. The presence of valvular heart disease can change the recommendations for embolism prophylaxis. Choice B, cardiac catheterization, is useful in patients suspected to have unstable angina, or who have sustained a myocardial infarction. Choice C, nuclear stress test, is useful in patients suspected to have angina pectoris, and may be a useful diagnostic study in this patient with cardiac risk factors (once the issue of atrial fibrillation has been treated). Choices D and E would be useful tests if the EKG had not established a diagnosis for this patient, with the Holter monitor indicated in patients experiencing symptoms on a daily basis, and the event recorder indicated in patients demonstrating more sporadic symptoms.

A 55-year-old woman with a history of hypertension and 2 vessel CABG presents to the emergency department with increasing dyspnea while walking up one flight of stairs. She denies chest pain and discomfort, but states that for the last 2 weeks she has also noticed palpitations. On physical examination, her vital signs are stable, with a normal physical exam. On EKG, she demonstrates atrial flutter with 2:1 AV block. Which of the following is the most appropriate next diagnostic study for this patient? A Transthoracic echocardiogram B Cardiac catheterization C Nuclear stress test D Holter monitor E Event recorder

The Correct Answer is: A Choice A, transthoracic echocardiogram, is the most appropriate next diagnostic study in this patient with atrial flutter, as it can demonstrate the presence of valvular heart disease. The presence of valvular heart disease can change the recommendations for embolism prophylaxis. Atrial flutter is treated similarly to atrial fibrillation in terms of embolism prophylaxis. Choice B, cardiac catheterization, is useful in patients suspected to have unstable angina, or who have sustained a myocardial infarction. Choice C, nuclear stress test, is useful in patients suspected to have angina pectoris, and may be a useful diagnostic study in this patient with cardiac risk factors once the issue of atrial fibrillation has been treated. Choices D and E would be useful tests if the EKG had not established a diagnosis for this patient, with the Holter monitor indicated in patients experiencing symptoms on a daily basis, and the event recorder indicated in patients demonstrating more sporadic symptoms.

A 40- year-old female patient comes to the office because she notices that she is easily fatigued but cannot pinpoint a direct cause of her fatigue. She has experienced muscle tenderness to the shoulders and other large muscle groups, sensitivity to touch these areas, and has also felt depressed lately. Based on this history and the vague physical exam findings, what is the most likely diagnosis? A Fibromyalgia B Polyarteritis nodosa C Polymyositis D Scleroderma E Sjogren's

The Correct Answer is: A Fibromyalgia syndrome (FMS) primarily affects woman between the ages of 20 and 60 and is now the second most common condition seen in rheumatologists' offices behind only rheumatoid arthritis. This condition spares the joints, but causes tender areas throughout the soft tissue as well as generalized pain and fatigue. There are several common trigger points, both anteriorly and posteriorly that help confirm the diagnosis. Polyarteritis nodosa is a systemic necrotizing vasculitis that generally affects medium size muscular arteries. Patients can present with many of the same symptoms as those with fibromyalgia (fatigue, muscle pain, and others), but these patients often have skin manifestations, joint pain, and fever, along with renal, GI, and cardiovascular signs not seen in fibromyalgia. Polymyositis is an inflammatory condition of the muscles that is characterized by proximal muscle weakness, with much less pain than is seen in fibromyalgia. Despite the proximal muscle weakness there is generally not significant atrophy. Sjogren's syndrome does not generally present with any muscular manifestations. It is a chronic inflammatory disorder that affects the salivary and lacrimal glands resulting in dry eyes and a dry mouth. Scleroderma is classically associated with thickened and hardened skin, but it can have internal organ involvement as well affecting many different body systems. When it affects the musculoskeletal system it tends to affect joints and the areas where tendons cross joints. It can cause contractures, pain, and swelling as well as fatigue and weakness.

A 23-year-old college basketball player twists her ankle while practicing. She explains the injury that is consistent with an inversion mechanism. Based on this history, what ligament would you expect to be the most likely injured in the ankle of this patient? A Anterior talofibular B Anterior tibiofibular C Calcaneofibular D Deltoid E Posterior talofibular ligament

The Correct Answer is: A Greater than 25,000 ankle sprains happen in the USA every day and the vast majority of those are inversion ankle sprains. The anterior talofibular ligament is the first, and often only, ligament damaged in inversion ankle sprains. As the force of the inversion increases, other lateral ankle ligaments can be involved. When the anterior tibiofibular ligament is involved, this is referred to as a high ankle sprain and such injuries generally have a prolonged recovery time. Calcaneofibular ligaments are generally the second most frequently injured of the lateral ankle ligaments and when injury occurs it is typically in combination with the anterior talofibular ligament. The deltoid ligament is a very strong ligament on the medial aspect of the ankle. Eversion stresses the deltoid ligament, but strong eversion forces are rare and when they do occur, an avulsion fracture of the medial malleolus is more likely than a significant ligament tear. The posterior talofibular ligament is one of the lateral ankle ligaments and can be injured in an inversion injury, but the rate of injury to this ligament lags far behind the anterior talofibular or calcaneofibular ligaments.

27-year-old female presents to the emergency department with severe RUQ pain. Ultrasonography reveals gallstones. In preparation for a potential cholecystectomy, a CBC is obtained that reveals a normocytic anemia with an increased mean corpuscular hemoglobin concentration (MCHC). She is slightly jaundiced and you are able to palpate her spleen on examination. What is her underlying diagnosis? A Hereditary spherocytosis B Iron deficiency anemia C Sickle cell anemia D Thalassemia E Von Willebrand's disease

The Correct Answer is: A Hereditary Spherocytosis (HS) is characterized by jaundice, an enlarged spleen, and often gallstones; gallstones are more frequently seen in young people, triggering the investigation into HS. An increased MCHC is a characteristic feature of HS and is almost the only condition in which this finding is seen. Iron deficiency anemia does not have an increased MCHC. Sickle Cell has characteristic findings different than the presenting findings. Thalassemia is noted for a microcytosis and Von Willebrand's is a coagulation disorder.

A 24-year-old man with a recent history of a viral illness comes to the emergency room complaining of severe left-sided chest discomfort, which radiates through to the left trapezius region. On coming into the room, you note that he is sitting up and hunched forward. On physical examination, the patient's temperature is 39°C, blood pressure is 135/78, with a pulse of 85 bpm, and a pericardial friction rub is noted. Laboratory findings demonstrate elevated serum creatine kinase levels and normal serial troponin levels. Which of the following would be the most likely electrocardiographic findings? A Diffuse ST segment elevation B Peaked T waves C Inferior Q waves D Loss of R-wave amplitude E U waves

The Correct Answer is: A In a patient with these signs, symptoms, and lab findings, acute pericarditis is the most likely diagnosis. In patients with acute pericarditis, EKG changes occur secondary to inflammation of the subepicardium, leading to widespread elevation of the ST segments, often with upward concavity, which returns to normal after several days, followed by T wave inversion. No significant QRS complex changes are noted, so choice C, the development of inferior Q waves (frequently associated with an inferior myocardial infarction), is incorrect. Choice B is frequently noted with severe hyperkalemia. Loss of R-wave amplitude, choice D, is associated with myocardial infarction. Choice E, U waves, are associated with hypokalemia

68-year-old woman with a history of hypertension and diabetes mellitus type 2 comes to the emergency department with her son, who noticed that while decorating for Christmas she seemed more dyspneic than normal, and had to sit down frequently. In addition, he noticed that she was pale and diaphoretic, and insisted on driving her to the emergency department. On questioning, she denies chest pain, but admits to being more fatigued than usual, with frequent jaw discomfort during activity. Activities such as vacuuming her house cause dyspnea, and she now has to stop several times while carrying laundry up from the basement. On physical examination, the patient's blood pressure is 90/50, pulse 99 bpm, respirations 22, and she is afebrile. Auscultation of the chest demonstrates a new systolic murmur. An EKG demonstrates normal sinus rhythm with nonspecific ST and T wave changes. Serial troponin elevations above the 99 th percentile of normal are noted. Which of the following would be the most appropriate next step in the management of this patient? A Clopidogrel, heparin, and aspirin, followed by cardiac catheterization B Nuclear stress test C Treadmill stress test D Thiazide diuretics and loop diuretics E Dobutamine stress echocardiogram

The Correct Answer is: A In patients with non-ST-segment myocardial infarction, such as this patient with ischemic symptoms and serial troponin elevation above the 99 th percentile of normal, clopidogrel, aspirin, and heparin prior to cardiac catheterization are recommended, with the intention of percutaneous coronary intervention. In addition, morphine, oxygen, nitrates, beta blockade, ACE inhibitors, statin therapy, and glycoprotein IIb/IIa inhibitors should be considered, depending on the patient's blood pressure, heart rate, and hemodynamic stability. Choice D would be useful in the treatment of patients with hypertension and lower extremity edema. Choices B, C, and E, are all forms of stress testing, which should be performed in patients with symptoms of angina pectoris, but not for patients with acute myocardial infarction.

50-year-old woman with a history of hypertension complains of chest tightness and dyspnea while walking up one flight of stairs. She recently experienced an episode of near-syncope while walking her dog. She denies a history of rheumatic fever. On auscultation, a crescendo-decrescendo systolic ejection murmur is heard at the upper right sternal border radiating to the carotids bilaterally. Troponin levels are negative at 0, 3, and 6 hours. Her EKG demonstrates evidence of left ventricular hypertrophy. A transthoracic echocardiogram reveals significant aortic stenosis secondary to congenital bicuspid aortic valve, left ventricular hypertrophy, and normal left ventricular systolic function. Which of the following is the most appropriate next step in management? A Cardiac catheterization followed by aortic valve replacement B Monitoring via repeat transthoracic echocardiogram in 6 months C Monitoring via transesophageal echocardiogram in 6 months D Treadmill exercise stress test E Automatic internal cardiac defibrillator placement

The Correct Answer is: A In symptomatic patients demonstrating significant aortic stenosis, aortic valve replacement after cardiac catheterization, to evaluate for coronary artery disease and possible concomitant coronary artery bypass surgery with aortic valve replacement, is indicated. As the patient is demonstrating the classic symptoms of severe aortic stenosis, choice A is the most appropriate next step in management. Choices B and C are thus inappropriate, as the patient is already symptomatic. If the patient were not symptomatic, choice B would be a viable choice compared to choice C, because it is less invasive than transesophageal echocardiogram. Choice D would be inappropriate, as strenuous physical activity should be avoided in patients with severe aortic stenosis. Choice E is appropriate therapy for patients at risk for ventricular tachycardia/fibrillation

A 36-year-old auto mechanic presents to the emergency department after hurting his back on the job. While lifting an object, he experienced sudden pain in his lower back with radiation to the right buttock. He was initially treated for muscle strain with a nonsteroidal anti-inflammatory drug (NSAID) after x-rays of his lumbosacral spine demonstrated no pathology. He continued to complain of this low back pain now radiating posteriorly down his left leg to the mid-thigh. Physical examination is unremarkable. The most likely diagnosis is A lumbosacral strain B left S1 radiculopathy C cauda equina syndrome D L5-S1 disc herniation E lateral femoral cutaneous neuropathy

The Correct Answer is: A Low back pain is one of the more common presenting neurologic complaints to a primary care provider. Most acute pain syndromes are benign, self-limiting conditions, with pain arising from myofascial sources. Patients with back pain and normal neurologic examinations are unlikely to have any serious underlying pathology and further diagnostic testing is usually unrevealing.

Which of the following is the drug of choice for acute hypertensive encephalopathy? A Labetolol B Clonidine C Furosemide D Nifedipine E Nitroglycerin

The Correct Answer is: A Neurologic emergencies associated with elevated blood pressure must be throurougly evaluated to determine the diagnosis and appropriate treatment plan. In the case of hypertensive encephalopathy, immediate attention must be focused on blood pressure reduction. Multiple agents are known to decrease blood pressure, but selection must also focus on how rapidly each agent works, how titratable the agent is, and any potential sequelae from using a particular agent. Sodium nitroprusside was classically the agent of choice for rapid blood pressure management, but it has fallen out of favor due to its monitoring requirements and rate of toxicity. Labetolol, a beta-blocker, is recommended for acute management, except in the case of cocaine intoxication. It has a rapid onset of action, can be titrated, and is given IV. Use of labetolol should be avoided in patients with known asthma, COPD, congestive heart failure, bradycardia, and second or third degree heart block. Additional agents appropriate for hypertensive encephalopathy include enalaprilat, esmolol, fenoldopam, hydralazine, and nicardipine, with each being considered carefully (based on patient condition and other factors). Agents with known CNS adverse effects, such as clonidine, should be avoided. Use of clonidine must also be monitored due to potential rebound hypertension. Nifedipine use is discouraged in hypertensive emergencies. Nitroglycerin should not be used for hypertensive encephalopathy because it increases intracranial pressure (additional information is available in Table 61-4).

22-year-old male hurts his right knee while playing football on artificial turf. He states that he planted his foot and went to turn, but his leg didn't turn with his body. He instantly felt a popping sensation in the knee. A few hours later he develops an effusion. Based on the history of the injury, which knee structure was likely injured? A Anterior cruciate ligament (ACL) B Lateral collateral ligament (LCL) C Medial collateral ligament (MCL) D Posterior cruciate ligament (PCL) E Quadriceps tendon

The Correct Answer is: A Non-impact rotational or hyperextension forces are the most common mechanisms for sustaining a tear of the ACL. One third of patients report hearing an audible popping sound as their ACL tear occurred. Because the ACL is a vascular structure, when it tears a rapid bloody effusion (hemarthrosis) usually develops which effects mobility of the joint. Lateral collateral ligaments are the least likely to be injured as the type of force necessary to cause injury would be a varus stress which is unlikely to occur in typical circumstances. Medial collateral ligament injuries are fairly common and produced by a valgus force that stresses the ligament. This can occur in many sporting events including those in which another competitor might fall on or dive into the lateral aspect of the knee. Trauma to a knee can result in tears of both the ACL and MCL in certain situations. A tear of the quadriceps tendon usually occurs when a person falls on a knee that is partially flexed. As the quadriceps muscle contract to prevent excessive flexion, the force and momentum of the fall may overwhelm the knee extension mechanism and cause the rupture. No such mechanism occurred in our scenario. Posterior cruciate injuries occur when the tibia is driven posterior in relation to the femur as may happen when a car dashboard is driven into the tibias during a major front impact collision. A powerful hyperextension force can result in both ACL and PCL tears (usually in that order). PCL tears are much more uncommon than ACL tears and don't generally occur with basic rotational forces as described in our patient scenario.

What age group is most at risk to develop osteoid osteoma? A Adolescents B Elderly C Middle-aged D Neonates E Toddlers

The Correct Answer is: A Osteoid osteoma is a benign bone forming tumor that usually develops during a patient's second decade of life. This type of tumor is much more common in boys than girls and typically affects the lower extremities (femur and tibia primarily) and spine more than other areas of the body. Patients typically present with gradually progressive bone pain that is worse at night and does not correlate with activity level. The tumor produces high levels of prostaglandins, so symptoms usually improve in 20-25 minutes if the patient takes a medication like ibuprofen, ASA or other NSAIDS that are prostaglandin inhibitors. A lack of improvement in symptoms with these medications should lead health care providers to consider a different diagnosis. The pain of this condition may cause those afflicted in a leg to limp and have swelling, muscle atrophy or contractures and exquisite point tenderness. The condition usually resolves on its own over time, but symptomatic patients may require surgical resection or radioablation of the tumor.

What is the most common blood gas abnormality in patients with a pulmonary embolism? A Respiratory alkalosis B Metabolic acidosis C Metabolic alkalosis D Respiratory acidosis E Compensated respiratory acidosis

The Correct Answer is: A Respiratory alkalosis occurs as a result of the hyperventilation.

A 22 year-old male is involved in a motor vehicle crash resulting in fracture of the left femur and left ribs 3 through 6. Approximately 24 to 36 hours after admission he becomes mildly confused and his RR increases to 40. Chest x-ray reveals diffuse pulmonary opacities. ABG shows pH 7.39, PCO 2 34, PO 2 55. What is the most likely diagnosis? A ARDS B Cardiac contusion C Pleural effusion D Pneumothorax E Pulmonary thromboembolism

The Correct Answer is: A The acute onset of respiratory distress after trauma is consistent with ARDS (A). The chest x-ray findings are inconsistent with cardiac contusion (B), pleural effusion (C), pneumothorax (D) and pulmonary thromboembolism (E). (Mention by letter)

Shoulder dislocation commonly involves injury to which nerve? A Axillary B Median C Peroneal D Radial E Ulnar

The Correct Answer is: A The axillary nerve is in close proximity to the glenohumeral joint, thus making it vulnerable to injury during a shoulder dislocation. The median, radial, ulnar nerves are more distal in the upper extremity and and the peroneal nerve is in the leg.

A 76-year-old man, is brought to the emergency department by his niece after she found him wandering around his yard in the cold wearing only a tee shirt and jeans. When she set up his pill container about 36 hours earlier, he seemed his usual self but, in retrospect, possibly a little more confused than usual. The niece says that he has "high blood," treated with a "white fluid pill," "sugar diabetes," treated with an oral medication, and early "old timer's" dementia treated with "a memory pill." Vital signs include an oral temperature of 100.8F, pulse 100 beats per minute, respirations 24 and somewhat shallow, and blood pressure of 88/52. Initial examination reveals a slightly dehydrated, stuporous man appearing older than his stated age, who smells strongly of urine. He has no lateralizing signs. What is the most likely cause of the mental status changes? A hyperglycemic hyperosmolar state B lactic acidosis C stroke D urinary tract infection E worsening dementia

The Correct Answer is: A The combination of confusion and dehydration in a patient with diabetes type 2 who is taking a diuretic strongly suggest hyperosmolar state. Patients with lactic acidosis (B) have marked hyperventilation and, usually, signs and symptoms of a serious illness. The lack of lateralizing signs makes a stroke (C) less likely. Urinary tract infection (D) could certainly cause confusion and incontinence in an elderly man and should be investigated. Alzheimer dementia (E) progresses slowly; sudden decompensation is usually due to delirium.

Upon testing a patient for function of the hip extensors, which muscle is considered the primary muscle responsible for most extension? A Gluteus maximus B Pectineus C Semimembranosus D Semitendinosus E Vastus lateralis

The Correct Answer is: A The gluteus maximus is a large muscle that is partially responsible for giving shape to the buttocks. It is the dominant muscle responsible for hip extension. It is easily palpable with a patient in the prone position with buttocks squeezed together or with the hip extended and the knee flexed. The pectineus muscle is considered a secondary hip adductor. The Semimembranosus and Semitendinosus are two of the three hamstring muscles. They are primary movers in knee flexion, but only secondary contributors to hip extension. Vastus lateralis is one of the four quadriceps muscles and plays a role in knee extension, but not hip extension.

A 32 year-old male presents to the emergency department in Acute Renal Insufficiency (AKI). Which of the following conditions would be most likely observed in intrinsic AKI? A Septic shock B Congestive heart failure C Benign prostatic hypertrophy D NSAID overdose E Chronic liver failure

The Correct Answer is: A The most common causes of intrinsic AKI are sepsis, ischemia, and nephrotoxins, both endogenous and exogenous. Prerenal acute kidney injury can be caused from hypovolemia, decreased cardiac output, decreased circulation of blood volume (CHF, liver failure), and impaired renal autoregulation (NSAIDs, ACE-I/ARB, cyclosporine)---(E), (D), and (B). Postrenal causes include bladder outlet obstruction including bladder stones and BPH (C).

32-year-old female is brought into the emergency department by her partner. His report indicates that she had been in her usual state of good health until a couple of days ago. At that time she started to complain of feeling fatigued. She now appears jaundiced and lethargic, and is complaining of chest pain. On exam, her spleen is palpable. Hemoglobin is 6 g/dl and she is Coombs positive. What is the most likely diagnosis? A Autoimmune hemolytic anemia B Glucose-6-phosphate dehydrogenase deficiency C Hereditary spherocytosis D Pyruvate kinase deficiency E Thalassemia

The Correct Answer is: A The onset of autoimmune hemolytic anemia (AHA) is often abrupt and dramatic. Anemia can develop in days, along with jaundice and splenic enlargement. When this triad is present, the suspicion for AHA must be high. The diagnostic test for AHA is the Coombs test. If positive, it confirms the presence of the antibody on the red cells. All other diseases listed are Coombs negative hemolytic anemias.

23-year-old man presents to the outpatient clinic for follow-up from a recent urgent care visit. He complains of sore throat, fever, fatigue, myalgias, and a rash that started 5 days ago, and have worsened since he was seen in the urgent care 3 days ago. The patient appears non-toxic with a temperature of 39.4 degrees Celsius. Physical exam reveals pharyngeal and tonsillar erythema without exudates, generalized lymphadenopathy, a morbilliform rash on his trunk, and no hepatosplenomegaly. A rapid strep screen and Monospot performed at the local urgent care were reportedly negative. Which of the following prevention strategies should be recommended to this patient? A Abstain from sexual activity B Avoid aspirin C Avoid contact sports and rest D Bedrest and increased fluids E Take the full course of antibiotics

The Correct Answer is: A The patient presentation is consistent with acute retroviral syndrome. The patient is highly contagious and should be counseled on strategies to prevent transmission of HIV to others (A). Aspirin use in viral syndromes (B) is associated with Reye's syndrome, but most often occurs in children with influenza or varicella. Avoiding contact sports (C) is appropriate patient education for a patient with infectious mononucleosis, and patients with group A strep pharyngitis should be instructed to take the full course of their antibiotics (E).

78-year-old woman with a medical history of diabetes and hypertension presents to the emergency department complaining of left hand weakness and slurred speech. Which of the following tests is most likely to determine the source of an arterial thrombus? A carotid ultrasound B CT of the brain C erythrocyte sedimentation rate D magnetic resonance angiography (MRA) of the vertebral arteries

The Correct Answer is: A The patient's symptoms are consistent with pathology arising from the anterior cerebral circulation including the carotid arteries. A CT should be ordered to rule out acute hemorrhage and an erythrocyte sedimentation rate may be useful if giant cell arteritis were suspected. An MRA of the vertebral arteries would likely show deficits but is not likely to demonstrate the etiologic location of this stroke

A 23-year-old patient who has recently been on a ski trip presents with pain to the right hand after sustaining a fall. It is difficult to move, and there is pain on movement of the first digit. Based on this history what ligament would the patient most likely have injured? A 1st MCP joint ulnar collateral ligament B 2nd MCP joint ulnar collateral ligament C 3rd MCP joint ulnar collateral ligament D 4th MCP joint ulnar collateral ligament E 5th MCP joint ulnar collateral ligament

The Correct Answer is: A The ulnar collateral ligament at the base of the thumb, or 1st MCP joint, is often injured in forced abduction, such as a fall while skiing or during other sporting activities. An injury to this ligament has traditionally been called Gamekeeper's Thumb, but the origin of this term referred to a more chronic injury sustained by English gamekeepers as a result of the way they killed rabbits using their hands. Any of the MCP joint ulnar collateral ligaments could be injured in a fall if the mechanism of injury creates significant forces on the ligaments, but the 1st MCP joint is far more commonly injured than the others mentioned above.

In the emergency department, you are asked to evaluate a 77-year-old man with a history of HTN who had a syncopal episode while chasing after his dog. He admits to recent episodes of chest discomfort, also associated with activity, as well as dyspnea at lower levels of activity including walking up one flight of stairs. On physical exam, a grade III/IV crescendo-decrescendo systolic ejection murmur can be heard best over the right upper sternal border. His EKG demonstrates NSR @ 80 bpm, with evidence of left ventricular hypertrophy. His troponin levels are negative for ischemia. What is the next most appropriate test or procedure? A Echocardiography B VQ scan C CT scan of the head D Serum D-dimer levels E MRI of the heart

The Correct Answer is: A This patient exhibits all the signs of progression of aortic stenosis, thus echocardiography is the next most appropriate test. A determination of severity can then be made, with possible cardiac catheterization if severe aortic stenosis is suspected, in preparation for surgical intervention if necessary. A VQ scan is appropriate if pulmonary embolism were suspected. A CT scan of the head could be considered if a head injury was suspected, but would not be the next step in the management of this patient. Serum D-dimer levels might be used to rule out pulmonary embolism, although it is a fairly nonspecific test. An MRI of the heart is not considered standard of care for aortic stenosis.

A 47-year-old male presents to the hospital with complaints of palpitations. He has a history of hypertension, for which he takes diltiazem. On exam he is alert, awake, and oriented. His blood pressure is 144/76, his pulse rate is 52, and his respiratory rate is 18. Lung sounds are clear, and cardiac is a regularly irregular rhythm. There is no peripheral edema noted, and a neurological exam is non-focal. The rhythm strip shown is produced. Based on these findings, what is the next step in the treatment of this patient? A Stop diltiazem B Cardiac catheterization C Permanent pacemaker insertion D Immediate cardiac surgery E Intravenous digoxin

The Correct Answer is: A This patient has a second-degree AV block, Mobitz Type I, or Wenckebach. In this case, putting the patient on a calcium channel blocker can increase the vagal tone and slow the rate down. The best intervention for this patient is to stop the diltiazem. If there is a suspicion of ischemia, then cardiac catheterization is warranted. There is no indication for a pacer or surgery.

A 1-year-old boy is brought to the emergency department by his parents, who state that the child refuses to walk or crawl and begins crying when they stand him. Swelling to his right knee is noted; it is also warm to the touch and pain response is noted. His parents state that it seemed to start a couple of days ago and has gotten worse. They don't recall a trauma, but state that he seems to bruise easily. The child's mother states that she also bruises easily. Vitals are as follows: Temp: 38.0°C, HR: 70, RR: 15. What laboratory finding would you expect? A Prolonged aPTT (activated partial thromboplastin time) B Prolonged bleeding time C Prolonged PT (prothrombin time) D Prolonged thrombin clotting time E Thrombocytopenia

The Correct Answer is: A This patient has hemophilia A. Patients with severe hemophilia A have a prolonged aPTT; all of the other tests should be within the normal range.

77-year-old female who was admitted to the hospital for acute coronary syndrome was found to have the rhythm strip shown when she arrived to the floor. Her initial vital signs were as follows: temperature is 99.0, pulse rate is 140, blood pressure is 100/65, and respiratory rate is 16. An initial bolus and infusion of amiodarone was started, with little success of slowing the rate or converting the patient's rhythm. About an hour later, the blood pressure dropped to 80/55 and she was becoming diaphoretic, with increased dyspnea and some mild chest discomfort. What is the next step in treating this patient? A Synchronized cardioversion B Increase dose of amiodarone C Infusion of magnesium sulfate D Intravenous metoprolol E Adenosine

The Correct Answer is: A This patient has unstable, sustained ventricular tachycardia. Because of the symptoms, and most importantly the blood pressure, it is critical to intervene immediately to prevent a cardiac arrest. The treatment option of choice in this case is synchronized cardioversion. Adenosine is not a viable option and may put the patient into a lethal arrhythmia.

What is the most likely reason for a patient to have cardiogenic shock? A Acute myocardial infarction B Sepsis C Trauma D Aortic dissection E Pericardial effusion

The Correct Answer is: A While trauma can account for a patient being subjected to cardiogenic shock, the overwhelming etiology for the shock comes from an MI.

A 62-year-old female with a known history of hypertension presents due to the abrupt onset of a severe headache, marked vertigo, nausea, vomiting, and ataxia. On physical exam, you note nystagmus, sensory loss, and weakness of the right face. What is the most likely diagnosis? A Cervical artery dissection B Cerebellar hemorrhage C Internal auditory artery occlusion D Lacunar infarction E Spinal cord compression

The Correct Answer is: B A cerebellar hemorrhage can result from multiple causes, similar to other intracerebral hemorrhages, such as hypertension, blood dyscrasias, trauma, and arteriovenous malformations. Patient presentation may range from an abrupt onset of headache to full coma, depending on the severity of the hemorrhage. Presentation is often similar to cerebellar infarction, with CT findings assisting in the differentiation. Symptoms may include all of those listed above as well as nuchal pain, altered consciousness, altered respiratory rate, abnormal eye movements, and impaired papillary responses. Cervical artery dissection may present with similar symptoms, including the abrupt onset of headache, but is not typically associated with ataxia, vertigo, or vomiting. Internal auditory artery occlusion is associated with vertigo and unilateral hearing loss. Lacunar infarction occurs within small resistance vasculature, with motor and sensory symptoms related to the impacted brain region. Spinal cord compression may be acute or progressive, with motor weakness and sensory loss caudad to the lesion.

A 75-year-old man with type 2 diabetes presents to the emergency department with a 2-day history of confusion and lethargy. On physical exam, notable dehydration, tachycardia, and confused mental state is noted. Serum sodium, potassium, magnesium, and chloride levels are normal. The arterial blood gases are normal and serum ketones are negative. The abnormal laboratory findings are as follows: Given this information, what is the most likely diagnosis? A diabetic ketoacidosis B hyperglycemic hyperosmolar state C hypoglycemia D dehydration

The Correct Answer is: B A hyperglycemic hyperosmolar state is characterized by dehydration, significant hyperglycemia, and an elevated serum osmolality with an insignificant or negative ketosis. Because of the lack of ketosis, the patient may present with a gradual onset of symptoms, and it can go unnoticed until the dehydration becomes more severe than in ketoacidosis.

A 35-year-old male presents with pain and decreased range of motion after sustaining a fall in which the patient tried to grab onto a bar which pulled his entire arm in the process. Given this clinical scenario, at what cervical motor neuron level would the biceps reflex be testing? A C4 B C5 C C6 D C7 E C8

The Correct Answer is: B C5 is the primary motor neuron being tested in a biceps reflex. C6 contributes to the brachioradialis reflex primarily, but does have a small role in the biceps reflex. The C7 motor neuron is primarily involved in the triceps reflex. C4 and C8 do not contribute to any primary reflexes.

A 22-year-old man is brought to the emergency department by paramedics after having sustained a single stab wound along the left sternal border at the fourth intercostal space. Upon arrival to the emergency department, he was hypotensive and tachycardic. The neck veins were distended and heart sounds were muffled. Which of the following interventions is the most appropriate first-line management of this patient? A Left tube thoracostomy B Pericardiocentesis C Fluid resuscitation D Immediate intubation

The Correct Answer is: B Cardiac tamponade is classically described by the triad of jugular venous distension (JVD), arterial hypotension, and muffled heart sounds. In the emergency department, suspicion of this clinically entity is usually confirmed by ultrasonography and is acutely treated by pericardiocentesis, which will be diagnostic, therapeutic, and buy time until a definitive procedure can be done. A left tube thoracostomy may be indicated in this patient but would not relieve symptoms. Fluid resuscitation though applied to all trauma patients would help stabilize the patient until more therapeutic interventions could be completed. Immediate intubation, even if indicated, would require a prophylactic tube thoracostomy to prevent the development of tension pneumothorax in the event of an unrecognized lung injury. Emergency thoracotomy will relieve the signs and symptoms associated with cardiac tamponade and allow for repair of any underlying cardiac injuries

65-year-old recent alcoholic comes to the emergency department with recent onset of dyspnea with exertion, 3 pillow orthopnea, lower extremity edema, and palpitations, in which he describes his heart as racing. Which of the following is the most appropriate treatment for his high-output congestive heart failure? A IV dextrose alone B IV thiamine C IV enalapril D IV dopamine E IV diltiazem

The Correct Answer is: B Choice B is the most appropriate treatment, as the patient is demonstrating high output congestive heart failure secondary to beriberi, or thiamine deficiency. In 50% of patients, IV thiamine administration, along with other vitamins and glucose, will resolve the patient's symptoms.

A 55-year-old woman with a history of mitral valve replacement and mitral stenosis (secondary to rheumatic heart disease) presents to the emergency department with increasing dyspnea while walking up one flight of stairs. She denies chest pain and discomfort, but states that for the past few weeks she has noticed palpitations. She also admits to lower extremity edema, which is new within the last week. On EKG, she demonstrates atrial flutter with 2:1 AV block. Her INRs have been therapeutic for the past 4 weeks. Which of the following is the most appropriate next step in treating this patient? A IV quinidine B IV ibutilide C IV vasotec D IV amiodarone E IV dopamine

The Correct Answer is: B Choice B, IV ibutilide, has been found to be most effective in converting atrial flutter to sinus rhythm out of all the choices listed. Choice A is contraindicated, as the atrial conduction may decrease to the point that 1:1 atrial to ventricular conduction can occur with the administration of class I antiarrhythmics. The ventricular rate can then increase to rates greater than 200 bpm, and hemodynamic collapse may occur. Choice C is useful for blood pressure control, but not for heart rate control. Choice D is useful for chronic atrial flutter heart rate management, or for helping to maintain sinus rhythm after cardioversion has occurred. Choice E is useful for pressor support, and not for heart rate control or conversion to normal sinus rhythm

A 55-year-old woman with a history of hypertension and 2 vessel CABG presents to the emergency department with increasing dyspnea while walking up one flight of stairs. She denies chest pain and discomfort, but states that for the last 24 hours she has also noticed palpitations. On physical examination, her vital signs are stable. On EKG, she demonstrates atrial flutter with 2:1 AV block. Her echocardiogram demonstrates normal LV systolic function and normal valvular function. Which of the following is the most appropriate therapy for this patient? A IV ibutilide after 4 weeks of anticoagulation with warfarin B IV ibutilide alone C IV quinidine after 4 weeks of anticoagulation with warfarin D IV quinidine alone E IV dopamine

The Correct Answer is: B Choice B, IV ibutilide, is the most appropriate choice for this patient. Therapy for patients with atrial flutter and atrial fibrillation is the same in regards to anticoagulation; therefore, in a patient with a CHADS2 score of 1 and with symptoms of less than 48 hours duration, cardioversion to normal sinus rhythm, whether chemically or electrically, is recommended. Out of all the choices listed, IV ibutilide has been found to be most effective in converting atrial flutter to sinus rhythm. Choices C and D are contraindicated, regardless of the type of anticoagulation paired with it, as quinidine is a class I antiarrhythmic. The atrial conduction may decrease to the point that 1:1 atrial to ventricular conduction can occur with the administration of class I antiarrhythmics. The ventricular rate can then increase to rates greater than 200 bpm, and hemodynamic collapse may occur. Choice E is useful for pressor support, which is not indicated in this patient who is quite stable

A 24-year-old HIV-positive man comes to the emergency department complaining of severe left-sided chest discomfort, which radiates through to the left trapezius region. On coming into the room, you note that he is sitting up and hunched forward. On physical examination, the patient's blood pressure is 135/78, with a pulse of 85 bpm, and a pericardial friction rub is noted. Laboratory findings demonstrate elevated serum creatine kinase levels and normal serial troponin levels. His EKG demonstrates peaked T waves. His CXR demonstrates a "water bottle" cardiac silhouette. Which of the following diagnostic studies would be considered the most appropriate next step in management of this patient? A Cardiac catheterization B Transthoracic echocardiography C CT of the thorax D VQ scan E Lower extremity venous doppler

The Correct Answer is: B Choice B, transthoracic echocardiography, would allow for monitoring of a patient with acute pericarditis, to determine if a pericardial effusion and/or cardiac tamponade develops. Choice A, cardiac catheterization, would be appropriate in a patient with acute myocardial infarction. Choice C, CT of the thorax, is not as effective or specific as transthoracic echocardiography for the development of a pericardial effusion and/or tamponade. Choices D and E would be appropriate diagnostic studies if pulmonary embolism is suspected, but not as the next step in management of this patient with pericarditis

A 67-year-old female presents to the emergency department with abdominal pain, bloating, inability to pass gas or stool, and vomiting. Which of the following, if present, would be a contraindication to placing a nasogastric tube? A Esophagitis B Esophageal strictures C Gastritis D Peptic ulcer disease E Gastroesophageal reflux disease

The Correct Answer is: B Contraindications to nasogastric tube placement include choanal atresia, significant facial trauma, basilar skull fracture, esophageal stricture or atresia, esophageal burn, zenker's diverticulum, recent surgery on the esophagus or stomach, or a history of gastrectomy or bariatric surgery.

A 44-year-old female is involved in a motor vehicle accident, during which she suffered blunt trauma to the left chest and abdomen from the car door. She presents via rescue with marked dyspnea, tachypnea, and an oxygen saturation of 87% on room air. You obtain the chest x-ray shown. Based on the following x-ray, what is the diagnosis? A Cor pulmonale B Diaphragmatic rupture C Hemothorax D Pericarditis E Pneumothorax

The Correct Answer is: B Diaphragmatic rupture is often the result of direct injury to the diaphragm or increased intra-abdominal or intrathoracic pressure. Patient symptoms are often the result of impaired lung expansion and decreased oxygenation. Additional symptoms may include bowel obstruction or other nonspecific bowel complaints. The chest x-ray shows elevation and irregularity of the left diaphragmatic border, with decreased left lung volume. Also of note is a widened mediastinum, which may suggest additional injury, including to the aorta. Lung markings extend through the lung space, which is not consistent with a hemothorax or pneumothorax. Evaluation for a pericardial effusion and/or pericarditis should be performed on this patient, based on the history, with evaluation including echocardiography and EKG. The typical x-ray result for pericardial effusion is termed a "water bottle" heart. However, this would be difficult to assess in the setting of a diaphragmatic rupture. Cor pulmonale is not associated with trauma, and is due to lung disease or pulmonary vascular disease

A 14-year-old boy presents to the emergency department with acute scrotal pain and vomiting for the past 2 hours. His left testicle is in extreme pain and he states the pain started while playing basketball in gym class. Which diagnostic test would help confirm your suspected diagnosis? A Transillumination B Doppler ultrasound C Urine cultures and sensitivity D Radionuclide imaging E Serum human chorionic gonadotropin levels

The Correct Answer is: B Doppler ultrasound (B) is the diagnostic imaging of choice to confirm testicular torsion, with radionuclide imaging (D) a distant second. Testicular torsion is time sensitive and MRI or more invasive imaging may lead to the delay of surgical intervention. Time is critical (< 6 hours) for the salvage of the affected testicle. Transillumination (A) is seen in hydrocele fluid. Urine cultures and sensitivity (C) can confirm urinary tract infections or sexually transmitted diseases causing epididymitis. If an intrascrotal tumor is suspected, serum tumor beta-human chorionic gonadotropin levels (E) should be obtained.

A 33-year-old IV drug user presents to the emergency department with pleuritic chest pain, cough, chills, diaphoresis, anorexia, and malaise. On physical exam, her temperature is 40°C, BP 98/55, P 115 bpm, and RR 22. No murmur could be appreciated. Two separate blood cultures are positive for S.aureus. An EKG, CXR, and transesophageal echocardiogram are ordered. Which of the following lesions is most likely to be seen on TEE in this patient? A Aortic valve vegetation B Tricuspid valve vegetation C Mitral valve vegetation D Left ventricular hypertrophy E Ventricular septal defect

The Correct Answer is: B In almost 50% of cases involving IV drug users, the only site of infection is the tricuspid valve, and most lesions are right-sided, so choice B is the most appropriate answer. Left ventricular hypertrophy, choice D, is seen in patients with a history of hypertension. Choice E, ventricular septal defect, is frequently associated with a holosystolic murmur

What is the most common joint dislocation in children? A Ankle B Elbow C Finger D Knee E Shoulder

The Correct Answer is: B In children the elbow is the most commonly dislocated joint and it is the third most common joint dislocation for adults. The shoulder and finger are dislocated more frequently than the elbow in adults. Knee dislocations at the femur-tibia joint are rare (patellofemoral dislocations are more common) and ankle dislocations are also relatively rare.

What is the most common ECG abnormality in patients with a pulmonary embolism (PE)? A Atrial fibrillation B Sinus tachycardia C Ventricular ectopy D Sinus bradycardia E High grade AV block

The Correct Answer is: B In most cases, sinus tachycardia is the only abnormality in patients with a PE. You may also find some ECGs that will have non-specific ST-T wave changes. Sinus bradycardia and AV blocks are not common findings that are associated with PE

70-year-old man, with a history of HTN and aortic valve replacement 3 months ago, presents with complaints of arthralgia, myalgia, anorexia, fatigue, and weight loss over the last month, with recent dyspnea on exertion and lower extremity edema. Vital signs are as follows: Temperature 38°C, BP 102/64, P 98, RR 20. On physical exam, a new high-pitched, blowing, decrescendo diastolic murmur is noted along the left lower sternal border. Two separate blood cultures are positive for S. aureus, and found to be methicillin-resistant. A transesophageal echocardiogram demonstrates a paravalvular abscess. Which of the following is the most appropriate therapy in the management of this patient? A IV vancomycin B IV vancomycin, IV gentamicin, and PO rifampin with surgical treatment C IV amphotericin plus flycytosine, and surgical treatment D Outpatient IV ceftriaxone E IV penicillin G

The Correct Answer is: B In patients with prosthetic valve infection with methicillin-resistant S. aureus, the treatment of choice is IV vancomycin for 6 to 8 weeks, plus IV or IM gentamicin for the initial 2 weeks secondary to nephrotoxicity, and PO rifampin for 6 to 8 weeks, with susceptibility to gentamicin determined before initiation of rifampin. Surgical therapy decreases mortality in patients with S. aureus endocarditis, from over 70% with medical therapy alone to 25%, and should be considered in patients with paravalvular abscesses and symptoms suggestive of moderate to severe refractory congestive heart failure. Therefore, choice B is the most appropriate next step in the management of this patient. Choice A does not offer sufficient coverage for methicillin-resistant S. aureus.

In addition to insulin and fluid replacement with 0.9% saline, which electrolyte is commonly infused in the type 2 diabetic patient who arrives in the emergency department in a hyperglycemic, hyperosmolar, nonketotic state? A bicarbonate B potassium C calcium D magnesium E sulfate

The Correct Answer is: B Insulin not only causes cellular uptake of glucose but also of potassium. Hypokalemia may develop when insulin is infused to correct either a hyperglycemic hyperosmolar state or a diabetic ketoacidosis. Hence, in order to avoid hypokalemia, potassium chloride can be added to a saline solution, as long as the serum potassium is not elevated.

A 66-year-old woman presents to the emergency department with a complaint of abdominal pain and distension for the past 3 days. Examination reveals a protuberant abdomen with diminished bowel sounds and tympany to percussion. Flat and upright abdominal radiographs reveal distended loops of bowel with prominent haustral markings. Which of the following etiologies is the most likely cause of the patient's condition? A Volvulus B Adenocarcinoma C Diverticular disease D Strangulated hernia E Adhesions

The Correct Answer is: B Large bowel obstructions are most commonly caused by an adenocarcinoma (65%). This is followed in decreasing incidence by diverticular scarring and volvulus. Adhesions are the most common cause of small-bowel obstruction but are rare as a cause of large bowel obstruction. The presence of haustral markings on radiographic evaluation helps differentiate between small and large bowel involvement.

A 22-year-old patient with sickle cell disease presents to the emergency department complaining of chest pain, fever, and non-productive cough. On physical exam his temperature is 100.6˚F, BP is 144/88, pulse is 110, respiratory rate is 24, and pulse oximetry is 84%. CBC shows a WBC of 11,500, hemoglobin of 8.3%, and hematocrit of 28%. What is the most likely diagnosis? A Acute bronchitis B Acute chest syndrome C Asthma D Bronchiectasis E Pneumothorax

The Correct Answer is: B Patients with sickle cell disease are prone to acute chest syndrome, resulting from sickling of cells within the lung that typically presents with chest pain, tachypnea, cough, fever, and oxygen desaturation. Acute bronchitis (A) and bronchiectasis (D) typically present with a productive cough without significant oxygen desaturation and anemia. Patients with asthma (C) and spontaneous pneumothorax (E) will be afebrile and have normal hemoglobin and hematocrit.

What term is used to describe the forward movement of one vertebral body on the vertebra below it, as shown in this figure? A Spina bifida B Spondylolisthesis C Spondylolysis D Sprengel's deformity E Sustentaculum tali

The Correct Answer is: B Spondylolisthesis is the forward movement of one vertebral body on the vertebra below. This most commonly occurs with L5 on S1 (more than 85% of cases) or L4 on L5. It is often due to a spondylosis (a bony defect) in the pars articularis as pictured above which is an acquired condition that may develop as a result of a stress fracture and it is not uncommon in children or adolescents. A small number of spondylolithesis cases are congenital. Spina bifida refers to a non-union of the vertebral arch at the spinous process, which is a congenital condition. Sprengel's deformity refers to a scapula that only partially descends from the neck to the thorax. The higher than normal position of the scapula results in a shortened appearance of the neck. The sustentaculum tali refers to an anatomical landmark in the foot that supports the talus and serves as an attachment for the spring ligament. It is part of the calcaneus bone.

A disk herniation that is putting pressure on the L5 nerve root may present with weakness of what muscle(s)? A Anterior tibialis B Extensor hallucis longus C Gastrocnemius-soleus D Iliopsoas E Peroneus longus and brevis

The Correct Answer is: B The extensor hallucis longus muscle's motor function is associated the L5 motor neuron, which also supplies the gluteus medius and extensor digitorum longus and brevis muscles. The anterior tibialis muscle is supplied by the L4 motor neuron. Nerves emanating from T12, L1, L2 and L3 supply the iliopsoas. Gastrocnemius, soleus and peroneus longus and brevis are all supplied by nerves coming from the S1 area. The plantar flexing gastrocnemius and soleus muscles also are supplied by S2

A 5-year-old male is being evaluated for an acute injury to the right ankle. On the x-ray of the ankle there is a distal tibia fracture that involves the separation of the growth plate, as well as a small non-displaced chip fracture of the metaphysis of the tibia. Based on these findings, what type of Salter-Harris fracture does this child have? A I B II C III D IV E V

The Correct Answer is: B The growth plate is the most fragile part of the bone prior to bone maturation and thus is usually the first structure disrupted when force is applied. Statistically, Type II fractures are most common - those that involve both the growth plate and a chip fracture of the metaphysis.

Upon testing a patient for function of the hip flexors, which muscle is considered the primary muscle responsible for most flexion? A Gracilis B Iliopsoas C Rectus femoris D Sartorius E Vastus intermedius

The Correct Answer is: B The iliopsoas muscle is the primary hip flexor muscle. It originates at T12 and L1-5 vertebrae and intervertebral disks as well as the iliac fossa of the pelvis and connects to the femur at the lesser trochanter. The gracilis muscle is considered a secondary hip adductor. Rectus femoris does help with hip flexion, but in a secondary role to the iliopsoas. Rectus femoris is also involved in knee extension. Sartorius is also involved in hip flexion, but in a secondary role. The vastus intermedius muscle is one of the four quadriceps muscles and is involved with knee extension and is not involved in hip flexion.

Which peripheral nerve is involved in the most common compression neuropathy in the upper extremity? A Axillary B Median C Radial D Sciatic nerve E Ulnar

The Correct Answer is: B The median nerve is commonly compressed as it passes through the carpal tunnel in the wrist. This syndrome is most often diagnosed in middle aged or pregnant female patients. The axillary nerve passes through the axilla and is often compressed when patients use crutches improperly and bear weight on the axillary area. The ulnar nerve is second only to the median nerve and can be compressed as it passes through the cubital tunnel at the elbow or as it passes through the humeral and ulnar heads of the flexor carpi radialis muscle. Compression of the radial nerve (and its branches) as it passes through the radial tunnel on the lateral side of the elbow is often confused with lateral epicondylitis. The sciatic nerve is associated with the lower extremities and pain is often elicited as a result of a lumbar disk herniation causing nerve root impingement.

A 19-year-old woman presents to the emergency department complaining of headache. The headaches are generalized and increasing in intensity. They have not responded to over-the-counter (OTC) medications. She complains of approximately 1 week of blurred vision, intermittent diplopia, and vague dizziness. Her medical history includes obesity and acne. She takes Accutane and oral contraceptives. She is found to have bilateral papilledema, visual acuity of 20/30 on physical examination, and a normal MRI of the brain. The next most appropriate step would be A CT scan of the head B lumbar puncture C therapy with high-dose prednisone D stat cerebral arteriogram E reassurance and follow-up in the office in 6 months

The Correct Answer is: B The presence of headache associated with papilledema raises the concern for a brain tumor. The MRI excluded a mass lesion, raising a strong suspicion of pseudotumor cerebri. This is also known as benign intracranial hypertension. It is not a benign condition, however, since it causes severe headache and may result in visual loss. It is particularly frequent in obese adolescent girls and young women. The etiology is unknown but may be associated with the use of oral contraceptives, vitamin A, and tetracycline. The presentation consists of headaches caused by an increase in intracranial pressure and blurring of vision. There may be diplopia, but the remainder of the neurologic examination is unremarkable. Papilledema is virtually always part of the presentation. The mental status is normal. The differential diagnosis includes venous sinus thrombosis, sarcoidosis, and tuberculosis or carcinomatous meningitis. The last two are excluded by lumbar puncture. An abnormal cerebrospinal fluid is not consistent with pseudotumor cerebri. The diagnosis is made by excluding mass lesions with CT scan or MRI and demonstrating markedly increased intracranial pressure by lumbar puncture. The treatment involves weight loss, diuretics, and steroids. Repeat lumbar punctures to remove cerebrospinal fluid and decrease intracranial pressure are effective.

A soft tissue neck x-ray of a patient who complains of a progressively worsening sore throat reveals this lateral film (see image). Based on these findings, what is the initial treatment of choice for this patient? A Endotracheal intubation B Intravenous steroids C Ribovirin injection D Incision and drainage E Cricoidotomy

The Correct Answer is: B This case of acute epiglottitis is treated with immediate intravenous steroids. Provided that the patient is able to maintain the airway and also keep oxygen saturation rates above 92%, the patient can improve with steroids and supportive care. Antiviral medications have little effect on the overall illness.

An 89-year-old female was found to have had an episode of syncope while at home. There was no history prior to the event, and the patient denies any chest pain prior to the episode occurring. On exam, the patient is alert, awake, and oriented, and is only complaining of mild shortness of breath. Her ECG is as shown. Which of the following is the best choice for treating this patient? A No treatment B Permanent pacemaker C Digitalis therapy D Ablation therapy E Beta blockers

The Correct Answer is: B This patient has a third-degree AV block that is symptomatic, with syncope and mild shortness of breath. The treatment for this patient is pacemaker insertion. Immediate insertion depends on the vital signs of the patient. If the patient's vital signs are stable, a transdermal pacer can be used until a scheduled pacer insertion within 24 hours. If the vital signs are not stable, immediate intervention with either central venous pacing wires or an immediate pacer insertion is warranted.

A 64-year-old man has been experiencing signs and symptoms compatible with diverticular disease for the past 3 weeks. He now presents to the emergency department malnourished with severe left-sided lower abdominal pain. After appropriate workup and hydration, he is taken to the operating room where a perforated sigmoid colon is discovered with gross contamination. What is the most appropriate surgical intervention at this time? A Left colectomy with primary anastomosis B Hartmann procedure C Proctocolectomy D Abdominoperineal resection E Low anterior resection

The Correct Answer is: B This vignette is consistent with an emergent resection in an unprepared patient. The most appropriate therapy for an acute perforation is a Hartmann procedure, which includes resection of the affected portion of the bowel, a temporary diverting colostomy, and oversewing of the distal rectal stump; the second stage of the procedure will involve taking down the colostomy with anastomosis to the rectal stump. A colectomy with a primary anastomosis should not be done when the bowel is unprepared due to the significant risk of infection and leakage of the bowel at the site of the anastomosis. Abdominoperineal resection is used in the treatment of malignant disease of the lower rectum. In this procedure, a permanent colostomy is created and the entire rectum, anal canal, and anus are removed. In the management of benign disease of the lower rectum, a proctocolectomy is appropriate to preserve anal function.

You are performing a mental status exam on a 19-year-old male patient who was brought to the Emergency Department because of bizarre behavior. As you are speaking with him he keeps repeating words that rhyme with words that either you or he say, but make no sense and are unrelated to your questions. How will you document this behavior in his chart? A Circumstantiality B Clanging C Flight of ideas D Perseveration E Tangentiality

The Correct Answer is: B Word association based on rhyme is called clang association (B) and may be seen in psychotic disorders. Circumstantiality (A) is a disturbance in fluency where the speaker meanders on many side topics before returning to the topic at hand. Flight of ideas (C) is a rapid transition from thought to thought, leading to the speaker losing track of the original idea. Perseveration (D) is a fixed focus returning again and again to the same thought. Tangentiality (E) refers to a disturbance in continuity where the speaker shifts from one thought to another that may be only vaguely related.

A 7-year-old male presents to the emergency department with right leg pain after falling off a swing at the school playground. Imaging of the right lower extremity shows the following fracture pattern: Which type of Salter-Harris Classification is observed? A Type I B Type II C Type III D Type IV E Type V

The Correct Answer is: C A Salter-Harris Type III fracture (C) involves a portion of the epiphysis only. Type I (A) is the entire epiphysis, Type II (B) is the entire epiphysis along with a portion of the metaphysis, Type IV (D) involves a portion of the epiphysis along with a portion of the metaphysis, and Type V (E) is a compression injury of the epiphyseal plate (nothing is "broken off").

A 2-year-old child presents to the emergency department via ambulance due to a seizure lasting approximately 2 minutes with jerking and somnolence. En route in the ambulance her vital signs are: temperature 39°C rectal; pulse 120/min; respirations 32/min; blood pressure 110/64 mm Hg. Upon further questioning, her mother claimed she had a runny nose yesterday. On physical examination, she is sleepy but arousable with negative Kernig and Brudzinski signs. Which of the following seizures is the MOST likely diagnosis? A absence seizure B complex partial seizure C febrile seizure D simple partial seizure

The Correct Answer is: C A febrile seizure is a brief (less than 15 minutes), generalized, symmetric, tonic-clonic seizure associated with a febrile illness (temperature greater than 38.8°C) without any central nervous system infection or neurologic cause. An absence (petit mal) seizure is a brief (2 to 25 seconds) loss of consciousness that can occur multiple times per day. There is no loss of tone, and frequently the only observable behaviors are staring or minor movements such as lip smacking and semipurposeful movements of the hands. There is no postictal period. Complex partial seizures (psychomotor) have varied symptoms including alterations in consciousness, unresponsiveness, and repetitive complex motor activities that are purposeless. Often, at the beginning of the attack, there is a psychoillusory phenomenon such as hallucinations, visual distortions, visceral sensations, or feelings of intense emotions. Simple partial seizures include focal motor, adversive, and somatosensory seizures. Manifestations of these seizures are varied including hallucinatory, psychoillusory, or complex emotional phenomena. Children will interact normally with their environment, with the exception of those limitations imposed by the seizure. Following the seizure (minutes to hours), there may be transient paralysis of the affected body part.

24-year-old intoxicated male presents to the emergency department after being in a fight. He was punched in the nose, and now has mild deformity of the nose and some epistaxis. An x-ray reveals a fractured nasal bone. During his physical exam, what must you look for in order to prevent permanent destruction of his nasal septum? A Orbital fracture B Posterior epistaxis C Septal hematoma D Facial fracture E Deviated septum

The Correct Answer is: C A septal hematoma can cause ischemic necrosis of the nasal septal cartilage if not identified and drained. A deviated septum can be expected with a nasal bone fracture, and must be addressed by the otolaryngologist. Excessive epistaxis that does not resolve with direct pressure and anterior packing may indicate a posterior bleed.

What types of connective tissue are injured in a sprain? A Bones and muscles B Fascia and joint capsules C Ligaments and joint capsules D Muscles and tendons E Tendons and bones

The Correct Answer is: C A sprain involves injury to those tissues that give support to joints - ligaments and joint capsules. Injury to muscles, tendons, and fascia would all be classified as a strain. Injuries to bone would be classified as a fracture

68-year-old female presents to the emergency department with signs and symptoms of an acute ischemic stroke. The initial CT scan is normal. Her blood pressure is 164/105. What is the most appropriate treatment for the blood pressure of this patient? A Atenolol PO B Clonidine PO C Close monitoring D Labetolol IV E Nicardipine IV

The Correct Answer is: C Aggressively lowering blood pressure may decrease blood flow to the ischemic tissue, thus decreasing the chances of recovery or increasing the risk of further infarction. In the setting of an acute ischemic stroke, blood pressure elevation should be monitored closely, with some elevation expected. This elevation is expected to decline without medication in the first few hours to days, but if elevation continues to a systolic blood pressure greater than 220mmHg, or mean arterial pressure greater than 120mmHg, medication is advised. Medications may include intravenous labetolol or nicardipine, with close monitoring of the patient. After the acute phase following a stroke, appropriate oral medications may be considered for outpatient hypertension management.

A 16-year-old girl is brought to the emergency department by ambulance after reportedly ingesting "a bottle of aspirin." Vital signs are temperature 37.8°C oral; pulse 94/min; respirations 30/min; blood pressure 100/68 mm Hg. What would you expect the blood gases to show that would confirm she had swallowed the aspirin? A anion gap metabolic acidosis with respiratory acidosis B nonanion gap metabolic acidosis with respiratory alkalosis C anion gap metabolic acidosis with respiratory alkalosis D nonanion gap metabolic acidosis with respiratory acidosis

The Correct Answer is: C An acute salicylate overdose (greater than 150 mg/kg) will produce symptoms of salicylate intoxication. Chronic salicylate intoxication occurs with ingestion of greater than 100 mg/kg/day for at least 2 days. Salicylates affect most organ systems, leading to various metabolic abnormalities. Because salicylates are a gastric irritant, symptoms of vomiting and diarrhea occur soon after the overdose, which may contribute to the development of dehydration. Salicylates stimulate the respiratory center leading to hyperventilation and hyperpnea resulting in respiratory alkalosis and compensatory alkaluria. A characteristic feature of salicylate intoxication is the coexistence of a respiratory alkalosis with a widened anion gap metabolic acidosis.

An 8-month-old female is diagnosed with respiratory syncytial virus bronchiolitis while in the emergency department. Which of the following strongly indicates a need for admission to the hospital and continued monitoring? A Age of 8 months B Birth at 38 weeks C Feeding difficulty with decreased oxygen saturation D Oxygen saturation of 96% E Respiratory rate of 45bpm

The Correct Answer is: C Brochiolitis patients must be considered at risk of developing severe disease and/or apnea when certain criteria are present, thus requiring admission. This includes, but may not be limited to, the following: birth <37 weeks gestation, age <12 weeks, witnessed apnea, underlying cardiopulmonary disease, immunodeficiency, tachypnea based on expected respiratory rate per age, decreased oral intake or feeding difficulty with associated decreased oxygen saturation, decreased oxygen saturation with varying ranges based on source (most being <95%), a history of previous intubation, and a caregiver ability to adequately provide care and monitoring.

33-year-old IV drug user presents to the emergency department with chills, diaphoresis, anorexia, and malaise. On physical exam, her temperature is 40°C, BP 98/55, P 115 bpm, and RR 22. Two separate blood cultures are positive for S.aureus. Which of the following physical exam findings would confirm a clinical diagnosis of infective endocarditis, according to the Duke criteria? A Increase in valvular regurgitation B Irregularly irregularly pulse C Osler's nodes D Buccal hemmorhages E Koplik spots

The Correct Answer is: C Choice C, Osler's nodes, confirms the clinical diagnosis of infective endocarditis, as it is a minor criteria. The Duke criteria for the clinical diagnosis of infective endocarditis requires the documentation of two major criteria, or one major criteria and three minor criteria, or five minor criteria. The patient demonstrates the presence of one major criteria (two separate blood cultures with typical microorganisms for infective endocarditis) and two minor criteria (fever greater than 38.0°C and predisposing condition of IV drug use). Only a new valvular regurgitation, not an increase or change in preexisting murmur, is considered sufficient to qualify as a major criteria, so choice A is incorrect. An irregularly irregular pulse, choice B, is commonly seen in patients with atrial fibrillation, not with infective endocarditis. Choice D, conjunctival hemorrhages, not buccal hemorrhages, are one of the minor criteria. Choice E, Koplik spots, are buccal lesions seen in patients infected with measles, whereas the presence of Roth's spots does fulfill one of the minor criteria

A 55-year-old woman with a history of emphysema, who is undergoing chemotherapy for lung cancer, comes to the emergency department complaining of a sudden increase in dyspnea, with exertion and fatigue. On physical exam, hypotension, pulsus paradoxus, and muffled heart sounds are noted. On transthoracic echocardiography, cardiac tamponade is noted, with over 200 mL of pericardial fluid described. Which of the following would be this patient's most likely electrocardiographic finding (Figure 7)? A Torsades de pointes B U waves C Electrical alternans with sinus tachycardia D Peaked T waves E Convex elevation of the J point

The Correct Answer is: C Choice C, electrical alternans with sinus tachycardia, a beat-to-beat alteration in one or more components of the ECG signal, is considered a specific sign of pericardial effusion, often with cardiac tamponade, as it represents the periodic swinging motion of the heart in the effusion at a frequency that is ½ the heart rate. Choice A, torsades de pointes, is a type of ventricular tachycardia frequently seen, and is associated with electrolyte disturbances or the use of certain types of antiarrhythmic drugs. Choice B, U waves, are associated with hypokalemia. Choice D is frequently noted with severe hyperkalemia. Choice E, convex elevation of the J point, is seen in patients suffering from hypothermia.

A 7-year-old is diagnosed with an acute case of hematogenous osteomyelitis accompanied with fever and leukocytosis. Based on your knowledge of the disease, which bone is most likely to present with the infection? A Feet B Hands C Long bones D Pelvis E Vertebrae

The Correct Answer is: C Fortunately, hematogenous osteomyelitis is not common in children, but when it does occur it primarily is found in the long bones. The femur, tibia and humerus are the most typical locations for osteomyelitis in children. The highly vascular metaphysis of long bones contribute to the potential for hematogenous spread of the implicated pathogen. Osteomyelitis can occur at any of the locations mentioned in the answer choices given, but at a significantly lower rate than in the long bones. The rate of occurrence at several selected locations is given below: Feet - 9% Femur 25% Hands - 6% Humerus - 13% Pelvis - 8% Radius/ulna - 6% Tibia/fibula - 28% Vertebrae - 2%

A 16-year-old high school boy presents to the emergency department 4 hours after sustaining an abrasion to his knee after a fall while rollerblading on the school playground. His school immunization record reveals that his last diphtheria, tetanus, and pertussis (DTaP) booster was administered at age 4. In this situation, which of the following is the MOST appropriate plan? A administer tetanus toxoid B administer adult tetanus and diphtheria toxoid (Td) C administer diphtheria, tetanus toxoid, and acellular pertussis (Tdap) vaccine D administer tetanus immune globulin

The Correct Answer is: C Generalized tetanus (lockjaw) is a neurologic disease caused by Clostridium tetani. Although any open wound is a potential source for contamination with C tetani, those with dirt, soil, feces, or saliva are at increased risk. Tetanus-prone wounds contain devitalized tissue, especially those caused by punctures, frostbite, crush injury, or burns. Recommendations for tetanus prophylaxis in a child with a laceration or abrasion depend upon the number of previous vaccinations, occurrence of last booster, type of wound (clean or tetanus-prone), and age of child. In this case, the patient is older than 7 years and had all of his previous immunizations; however, his most recent booster was greater than 10 years ago. Thus, he should receive an adult-type diphtheria and tetanus toxoid with acellular pertussis. In most cases, when tetanus toxoid is required for wound prophylaxis in a child older than 7 years, the Td instead of tetanus toxoid alone is recommended so that diphtheria immunity is maintained. If tetanus immunization is not up to date at the time of wound treatment, then the immunization series should be completed according to the primary immunization schedule. If a child is younger than 7 years, then the diphtheria, tetanus, acellular pertussis (DTaP) booster is indicated, unless there is a contraindication for pertussis, in which case the diphtheria and tetanus (DT) booster should be administered. Tetanus immune globulin (TIG) is recommended for treatment of tetanus. Under special circumstances, a patient infected with the human immunodeficiency virus (HIV) with a tetanus-prone wound should also receive TIG in addition to the prophylactic vaccine. (Ogle and Anderson, 2009, pp. 1144-1147; Centers for Disease Control and Prevention, 2009a) Ogle JW , Anderson MS. Infections: bacterial & spirochetal. In: Hays WW , Levin MJ , Sondheimer JM, et al., eds. Current Diagnosis & Treatment: Pediatrics. 19th ed. New York: McGraw-Hill; 2009. Centers for Disease Control and Prevention (2009a). Summary of recommendations for tetanus toxoid, reduced diphtheria toxoid and acellular pertussis vaccine (Tdap) and tetanus and diphtheria toxoids (Td) use among adolescents aged 11-18 years .

A 25-year-old male has a history of schizophrenia, and was brought to the emergency department by ambulance after he was found wandering along the highway. Which of the following is considered a positive symptom of schizophrenia? A Catatonia B Diminished sociability C Hallucinations D Poverty of speech E Restricted affect

The Correct Answer is: C Hallucinations, delusions, and formal thought disorders are classified as positive symptoms of schizophrenia. Negative symptoms of schizophrenia include restricted affect, diminished sociability, and poverty of speech. Catatonia describes a state in which a person is withdrawn and shows minimal bodily movement, or one in which there is severe excitement and the person shows purposeless and stereotyped movements.

A 70-year-old woman who was found barely responsive at home by her daughter is brought to the emergency department. Evaluation reveals that she is in a hyperglycemic hyperosmolar state with a severe fluid deficit. Treatment is initiated with vigorous saline rehydration and a continuous infusion of insulin. At what point should her glucose be added to her treatment? A when her condition becomes stable B when her urine output reaches 50 mL/hour C when her blood glucose reaches 250 mg/dL D if she develops hypokalemia E if she begins to spill ketones in her urine

The Correct Answer is: C In hyperglycemic hyperosmolar states, the serum glucose rapidly corrects with fluid administration alone. However, with vigorous rehydration, glucose may fall precipitously and lead to severe hypoglycemia. To avoid this, glucose should be added to water, half-normal, or normal saline as soon as the patient's blood glucose is less than or equal to 250mg/dL. She should continue to receive insulin IV until she is stabilized (A) when it can be switched to subcutaneous administration. The goal of fluid therapy in this patient is restoring her urine output to 50 mL per hour (B) or more. Because insulin drives potassium into the cells and can cause hypokalemia (D), potassium chloride should be given unless the patient has chronic kidney disease or oliguria. Persons in a hyperglycemic hyperosmolar state typically do not spill ketones (E) the way persons with diabetic ketoacidosis do.

Which type(s) of Salter-Harris fractures can generally be treated with closed reduction and cast immobilization? A Type I B Types I and II C Types I, II, and III D Types I, II, III, and IV E Types I, II, III, IV, and V

The Correct Answer is: C Minimally displaced Salter-Harris types I, II, and III fractures generally can be treated with immobilization only. Types IV and V involve the cartilage of both the articular surface and the growth plate. To ensure proper alignment and a congruous joint surfaces open reduction and internal fixation is usually necessary.

An 18-year-old woman is transferred to your emergency department from a local college infirmary. She presented yesterday with a complaint of headache but became confused and is now febrile. You notice a petechial rash on physical examination and her cerebrospinal fluid comes back with increased WBCs, increased protein, and decreased glucose. What is the most likely organism responsible for her meningitis? A Haemophilus influenzae B cytomegalovirus C Neisseria meningitidis D Mycobacterium tuberculosis E coxsackievirus B

The Correct Answer is: C Neisseria meningitidis and Streptococcus pneumoniae are the most common etiologic agents for bacterial meningitis in this patient's age group. So much so that many colleges and universities require a vaccine for students who live in dormitories. Her fever and the cerebrospinal fluid values are consistent with a bacterial and not a viral infectious source for the meningeal irritation.

A 21 year-old female presents to the emergency department after having a cast applied to her right arm earlier that day. Approximately one hour ago she began having extreme 10/10 pain in her right arm and is in visible distress. When considering the diagnosis of compartment syndrome, permanent damage to the muscle begins after how many hours of ischemia? A >2 hours B >6 hours C >8 hours D >12 hours E >24 hours

The Correct Answer is: C Permanent damage results after >8 hours (C) of ischemia. Nerves begin to lose conduction within 2 hours of onset of elevated pressures. Neurapraxia can occur within 4 hours, and irreversible damage occurs 8 hours after elevated pressures. Functional impairment is unlikely when compartment syndrome is diagnosed and treated within 6 hours of its onset. While deficits can occur prior to 8 hours, permanent damage is usually not seen before 8 hours (A and B). After 12 or 24 hours, permanent damage has already resulted in most cases (D and E). The key to early detection of compartment syndrome is a high index of clinical suspicion.

What is the most common cause of pneumothorax in a healthy patient? A Traumatic B Infectious C Ruptured bleb D Surfactant abnormality E Malignancy

The Correct Answer is: C Rupture of a bleb is thought to be more relevant to young, thin men, and also patients who have a family history of them, and smoking.

A 4 year-old male presents to the emergency department with vomiting, severe abdominal pain, and diarrhea that occurred 2 hours after "getting candy from Grandma's purse". Which of the following medications did the child most likely ingest? A Cisplatin B Erythropoietin C Ferrous sulfate D Lisinopril E Vincristine

The Correct Answer is: C Stage 1 of iron toxicity secondary to ferrous sulfate ingestion (C) is characterized by acute GI irritation; this is followed by a latent phase (Stage 2) that can then progress to systemic iron toxicity (Stage 3), that can progress to hepatic failure (Stage 4) or delayed sequelae (Stage 5). Cisplatin (A), erythropoietin (B), and vincristine (E) are parenterally administered and unlikely to be present in a patient's purse. Lisinopril (D) toxicity consists of cardiovascular symptomatology (e.g., hypotension and tachycardia).

27-year-old African American with sickle cell anemia presents to the emergency department with acute onset intractable pain. She is taking quick, shallow breaths and her oxygen saturation is 84% on room air. She appears desiccated, states she hasn't eaten in the last 24 hours, and says that she "just doesn't feel well." She is also afebrile. What should your next course of action be? A Start morphine, hydrate, and start antibiotics B Start morphine, oxygen, and start antibiotics C Start oxygen, hydrate, and exchange transfusion D Start oxygen, hydrate, and give pneumococcal vaccination E Start oxygen, hydrate, and start antibiotics

The Correct Answer is: C Start oxygen, hydrate, and exchange transfusion Exchange transfusions are primarily indicated for the treatment of intractable pain crises, priapism, and stroke. Patients should be kept well hydrated, and oxygen should be given if the patient is hypoxic. Antibiotics would be used if there was an infection identified, but are not part of initial treatment in a sickle crises.

Radiculopathy due to nerve root compression occurs most commonly at which nerve root within the brachial plexus? A C5 B C6 C C7 D C8 E T1

The Correct Answer is: C The C7 nerve root is affected the most often (approximately 45-60%). This radiculopathy can result from foraminal encroachment of the spinal nerve, cervical disk herniation, tumor, and multiple sclerosis. C7 radiculopathy can present with weakness in the triceps, which cause elbow extension, and finger flexion and extension. C6 is another common site of radiculopathy. C6 radiculopathy can present with weakness in the biceps, brachioradialis, and wrist extensor muscles. Cervical radiculopathy at the C5, C8, and T1 are less common, but still possible. C5 radiculopathy can present with deltoid and biceps muscle weakness. C8 radiculopathy can present with finger flexor weakness and T1 radiculopathy with finger abduction weakness.

What is the initial treatment for a patient who is having an acute episode of supraventricular tachycardia? A Caffeine B Beta blockers C Valsava maneuver D No treatment E Synchronized cardioversion

The Correct Answer is: C The initial treatment that should be attempted is a simple vagal maneuver to break the reentry tachycardia. If this is unsuccessful, pharmacologic therapy is indicated.

A 21-year-old man presents to the emergency department complaining of a swollen left ear (see image) after he experienced blunt trauma in a collegiate wrestling match. Which of the following is the most appropriate management for this condition? A Evacuation B Evacuation followed by antibiotics C Evacuation with subsequent antibiotics and splinting D Ice and compression E Reevaluation in 24 hours

The Correct Answer is: C The patient has an auricular hematoma that must be evacuated to prevent cartilage necrosis or infection. Evacuation, antibiotics, and splinting (C) remove the hematoma, minimize the risk of infection, and help prevent the re-accumulation of blood.

3 year-old boy is brought to the emergency department due to acute onset of cough and wheezing. Physical exam reveals focal wheezing in the right lower lobe. Which of the following is the most effective treatment option for the patient's suspected diagnosis? A Albuterol B Azithromycin C Bronchoscopy D Chest physiotherapy E Prednisone

The Correct Answer is: C The patient has most likely aspirated a foreign body. The most effective treatment is removal of the foreign body through bronchoscopy (C). Supportive therapy includes bronchodilators for wheezing/airway obstruction (A), antibiotics for associated infections (B), and chest physiotherapy (D) to promote recovery after removal of the foreign body.

17-year-old boy high school wrestler is brought into the emergency department after he collapsed at a wrestling match. He spent time fully clothed in a hot sauna prior to the match to try to "make weight." Labs are ordered, and results come back as follows: Which IV fluid regimen would most effectively treat this patient's hypernatremia? A quarter normal (hypotonic) saline B half-normal saline C isotonic (normal) saline D dextrose 5% in water E lactated Ringer's

The Correct Answer is: C The patient presents with a combination of inadequate fluid intake and excessive losses due to perspiration, resulting in hypovolemia and hypernatremia. The most common causes of hypernatremia are inadequate fluid intake resulting in hemoconcentration and diabetes insipidus (DI), resulting in excessive renal fluid losses. Normal urine osmolality is 500 to 850 mOsm/kg but can range from 50 to 1,200 mOsm/kg depending on the patient's fluid intake. Urine osmolality >400 mOsm/kg indicates that the renal fluid-conserving mechanism is intact, as the kidneys are working to preserve volume. A lower urine osmolality would be consistent with DI, characterized by a lack of response to anti-diuretic hormone (ADH), resulting in excessive urinary losses of water with worsening hypernatremia. Treatment is directed at the cause. If the patient is dehydrated, restoring fluid volume is the goal. If the patient has DI, treating the underlying disease will lower the serum sodium level. For this dehydrated patient, the treatment would be to administer isotonic (normal) saline, which contains 0.9% sodium, because of the large free water deficit. Quarter-normal saline contains 0.25% sodium, half-normal saline contains 0.45% sodium, and lactated Ringer's solution is similar to half-normal saline in its sodium content. Dextrose 5% in water (D5W) contains no electrolytes. Isotonic saline is the appropriate choice because it treats not only the volume deficit but the serum osmolality as well. Its osmolality is often lower than the plasma osmolality because of the hypovolemic state and, therefore, helps restore normal serum osmolality. Once serum osmolality becomes more normal, the isotonic saline can be replaced by D5W to replace the remaining free water deficit. If the free water deficit were less dramatic, initial IV fluid treatment could be half-normal saline, followed by D5W

43-year-old male presents to the Emergency Department complaining of right eye pain after treating his yard with fertilizer and lime. He attempted to flush his eye at home without relief of pain. Which of the following is the most appropriate initial step in managing this patient's symptoms? A Double evert his eyelids to look for remaining foreign bodies B Fluorescein stain his eye C Instill proparacaine 0.5% ophthalmic solution D Irrigate his eye until the pH is between 6.8 and 7.4 E Refer to ophthalomogist

The Correct Answer is: C The patient requires all of the above steps and should be given pain relief (C) prior to thoroughly flushing the eye (D), removing foreign bodies (A), assessing for corneal injuries (B), and referring to ophthalmology (E).

A 62-year-old man is brought to the emergency department after being found unresponsive in his car. On physical examination, his pupils are noted to be 7 mm on the right and 3 mm on the left. Which of the following diagnostic tests is most likely to identify the cause of the patient's signs and symptoms? A CBC with differential B serum electrolytes C MRI with contrast D liver function tests E skull X-rays

The Correct Answer is: C The patient's unilateral symptoms are best explained by a local anatomical cause (e.g., tumor) that would be detected with an imaging study (MRI). An MRI is preferred over skull X-rays to assess directly for intracranial pathology. CNS abnormalities arising from systemic causes are more likely to be symmetric

Upon testing a patient for function of the hip abductors, which muscle is considered the primary muscle responsible for most abduction? A Biceps femoris B Gluteus maximus C Gluteus medius D Gluteus minimus E Vastus medialis

The Correct Answer is: C The primary mover in the motion of hip abduction is the gluteus medius muscle. Gluteus minimus does play a supporting role in that motion. Biceps femoris is one of the three hamstring muscles and contributes to the motions of knee flexion (primary muscle) and hip extension (secondary muscle). Gluteus maximus is the primary mover for hip extension and vastus medialis is one of the four quadriceps muscles responsible for knee extension, but no hip movements.

62-year-old man with a history of hypertension, diabetes mellitus type 2, hyperlipidemia, and chronic tobacco use presents to the emergency department with complaints of retrosternal chest pressure associated with diaphoresis, nausea, and dyspnea, radiating down his left arm for the last 45 minutes after mowing his lawn. His EKG is shown (Figure 3). Which of the following is the most likely diagnosis given this patient's history and EKG findings? A Acute pulmonary embolism B Esophageal spasm C Acute myocardial infarction D Costochondritis E Aortic dissection

The Correct Answer is: C This patient is demonstrating evidence of acute anterolateral myocardial infarction on EKG, with ST segment elevation across the precordial leads, and reciprocal changes in the inferior leads, indicative of left anterior descending coronary artery stenosis. In addition, this patient's history increases the risk of acute myocardial infarction; therefore, choice C is the most likely diagnosis. Acute pulmonary embolism may also present with chest discomfort, and the patient's history of chronic tobacco use does increase this patient's risk of a hypercoaguable state. However, EKG changes with pulmonary embolism differ from those pictured, in that usually on an EKG, a pulmonary embolism presents with a prominent S wave in lead I, Q wave in lead III, and inverted T wave in III (S1/Q3/T3). Choices B, D, and E may also present with retrosternal chest discomfort; however, no EKG changes would be noted.

A 27-year-old man presents to the emergency department with a five-day complaint of substernal pleuritic chest pain, which worsens while lying supine. He is in no distress. A friction rub is noted over the precordium. The patient's vital signs are as follows: temperature is 100.4˚F, pulse rate is 94, respiratory rate is 20, and blood pressure is 136/84. An ECG reveals widespread diffuse ST elevations with PR interval depressions. He was recently treated for a viral respiratory infection. Which of the following is the most appropriate initial management for this patient? A Administer a broad spectrum antibiotic B Administer intravenous tissue plasminogen activator (T-PA) C Begin a nonsteroidal anti-inflammatory agent D Perform needle thoracentesis E Refer for immediate cardiac catheterization

The Correct Answer is: C This patient's diagnosis is acute inflammatory pericarditis. Viral infections are the most common cause of acute pericarditis, and males are the most commonly affected. A pericardial friction rub and EKG changes are characteristic of this diagnosis. Treatment is focused on the underlying inflammation, with NSAIDS being first-line and short course corticosteroids also being appropriate. Antibiotics are not indicated unless a bacterial etiology is confirmed or there are significant risk factors

17-year-old male is brought to your Emergency Department by his girlfriend. She states that he has been behaving strangely for the last three days, with rapidly fluctuating moods ranging from euphoric to irritable and paranoid. The patient states that he is fine, just a little nervous about an upcoming test in school. His pulse is 126 beats per minute, BP 182/106, pupils are widely dilated, and he is diaphoretic. What is his most likely diagnosis? A Acute anxiety B Bipolar disorder C Cocaine intoxication D Heroin intoxication E Thyrotoxicosis

The Correct Answer is: C This patient's presentation with tachycardia, hypertension, diaphoresis, and mydriasis along with the behavioral changes is consistent with cocaine intoxication (C). His physical symptoms could be explained by thyrotoxicosis (E), but he has no history of hyperthyroid symptoms such as weight loss and it would not explain the mydriasis. Acute anxiety (A) or bipolar disorder (B) might explain some of the mood changes but would not account for all of his physical signs. Heroin withdrawal (D) would cause some similar symptoms, but intoxication would cause somnolence and pinpoint pupils

What types of connective tissue are injured in a strain? A Bones and muscles B Fascia and joint capsules C Ligaments and joint capsules D Muscles and tendons E Tendons and bones

The Correct Answer is: D A strain involves injury to the muscles and tendons that are responsible for active movement of various body parts. Fascia is a part of the muscle-tendon unit, so injury to fascia would be considered a strain as well. Injury to ligaments and joint capsules would be considered a sprain and damage to bone would be classified as a fracture.

An 8-year-old girl is rushed to the emergency department by her parents because she has become delirious. The child was diagnosed with influenza three days prior. Her parents say that she had begun vomiting yesterday, almost nonstop, and has not been able to hold down fluids. They also note that she has been breathing rapidly. Your exam reveals a tachypneic, disoriented female with hyperreflexia, a positive babinski reflex, and liver enlargement. CSF analysis reveals a normal protein and cell count. What is the most likely diagnosis? A Bacterial meningitis B Guillain Barre syndrome C Measles encephalitis D Reye's syndrome E Viral meningitis

The Correct Answer is: D Although rare, Reye's syndrome is associated with viral infections, salicylate use during illness, and metabolic disorders. Illness is associated with liver fat deposition and degeneration, intractable vomiting, and mental status changes, which may progress to seizures, delirium, and coma. Cerebral edema contributes to these changes and other neurologic findings. Meningeal signs are more consistent with meningitis. Measles encephalitis typically presents days to weeks after the pathognomic measles exanthem and clinical findings. Guillain Barre has been associated with influenza infection, and signs and symptoms would include evolving weakness with ascending paralysis and extremity dysesthesias

An avulsion fracture at the base of the fifth metatarsal is commonly called which of the following? A Bennett fracture B Boxer's fracture C Chauffer's fracture D Jones fracture E Lisfranc fracture

The Correct Answer is: D An avulsion fracture at the base of the fifth metatarsal, usually secondary to plantar flexion and inversion is called a Jones fracture. Also called a ballet or dancer's fracture, it is the most common metatarsal fracture. The fracture occurs at the proximal diaphysis. A Bennett fracture is an oblique fracture of the first metacarpal near the carpometacarpal joint. A boxer's fracture is a fracture of the fifth metacarpal. This is the most common fracture of the hand. A chauffer's fracture is an oblique fracture through the base of the radial styloid in the forearm. A Lisfranc fracture is actually a fracture and dislocation involving the tarsometatarsal joints.

16-year-old male soccer player is complaining of pain to the right foot that has been getting progressively worse for the last 2 months. He states it hurts the most when he has all of his weight on his right foot as he plants to kick the ball. Most of the pain appears to be on weight bearing. You are concerned that this patient may be developing a stress fracture. Based on the patient's history and patient presentations, which bone is the most affected by stress fractures in the foot? A Calcaneus B Fifth metatarsal C First metatarsal D Second metatarsal E Talus

The Correct Answer is: D Any bone that is exposed to repetitive stress can have a stress fracture, but the long and thin metatarsal bones of the foot are the most commonly affected bones. Of the metatarsals, the second metatarsal has the highest number of stress fractures. These weight bearing bones can be particularly vulnerable to stress fracture if the patient is involved in long distance running, especially if he/she is wearing improper footwear for that activity or footwear that has lost most of its shock absorbing abilities. Some young female athletes may be training so hard that they become amenorrheic which can contribute to osteopenia resulting in weaker bones. Older patients with osteoporosis will also have a higher risk of stress fracture. Initially stress fractures of the metatarsals may present with a small area of localized pain and the dorsal forefoot may demonstrate a fairly diffuse area of swelling. If the stress fracture is not treated early, some patients will experience an audible pop or crack as the incomplete stress fractures progresses to a complete break. All types of fractures occur more easily in long thin bones like the metatarsals, than thicker bones like the calcaneus and talus

What nerve is most commonly injured in a mid- or distal humeral shaft fracture? A Axillary B Median C Peroneal D Radial E Ulnar

The Correct Answer is: D Because of the radial nerves proximity to the humerus, mid and distal shaft fractures with significant displacement can cause a radial nerve injury. Median and ulnar injuries are more commonly associated with forearm injuries. Axillary nerve injuries are most common in anterior shoulder dislocations and peroneal nerve damage occurs as a result of lower leg insult.

A 42-year-old woman with a history of migraine cephalgia and Raynaud's phenomenon comes to the emergency department with complaints of severe chest discomfort that occurs at rest every morning (at approximately 10 AM). An EKG performed during an episode of chest discomfort demonstrates transient ST segment elevation, which is relieved with sublingual nitroglycerin. There is no troponin elevation. Cardiac catheterization is performed, and reveals coronary artery spasm, which corresponds with ST segment elevation, and no significant coronary artery stenosis. Which of the following is the most appropriate treatment regimen? A 24-hour nitroglycerin dermal patch B Thiazide diuretics C Loop diuretics D Calcium channel blockers E Aspirin

The Correct Answer is: D Choice D, calcium channel blockers, and long-acting nitrate therapy have been proven to be effective for preventing recurrences of episodes of Prinzmetal angina, with short-acting sublingual or IV nitroglycerin useful for relieving acute episodes. Choice A, 24-hour nitroglycerin dermal patch, is inappropriate, as patients can develop nitrate tolerance, and thus need a 12-hour nitrate-free period every day. Choices B and C, thiazide and loop diuretics, have no proven benefit in patients with Prinzmetal angina. Aspirin, choice E, may worsen episodes of prinzmetal angina, and thus is not recommended

A 68-year-old woman with a history of hypertension and diabetes mellitus type 2 comes to the emergency department with her son, who noticed that while decorating for Christmas she seemed more dyspneic than normal, and had to sit down frequently. In addition, he noticed that she was pale and diaphoretic, and insisted on driving her to the emergency department. On questioning, she denies chest pain, but admits to being more fatigued than usual, with frequent jaw discomfort during activity. Activities such as vacuuming her house cause dyspnea, and she now has to stop several times while carrying laundry up from the basement. On physical examination, the patient's blood pressure is 90/50, pulse 99 bpm, respirations 22, and she is afebrile. Auscultation of the chest demonstrates a new systolic murmur. An EKG demonstrates normal sinus rhythm with nonspecific ST and T wave changes. Which of the following would be the most appropriate next step in the management of this patient? A Transesophageal echocardiogram B Nuclear stress test C Cardiac catheterization D Serial serum troponin levels E CXR

The Correct Answer is: D Choice D, checking serial serum troponin levels, is the most appropriate next step in the management of this patient. Women and diabetics may present with atypical symptoms with acute non-ST-segment myocardial infarction, including dyspnea, jaw discomfort, and epigastric discomfort. Frequently, women present much later than men with these symptoms. Therefore, a high level of suspicion should be maintained when women present with symptoms of dyspnea, even in the setting of nonspecific EKG changes, and drawing serum troponin levels before any other testing is recommended. Once non-ST-segment myocardial infarction has been ruled out, choices E and B, and also transthoracic echocardiogram, would likely be evaluated. Transesophageal echocardiogram may be required if better visualization of the heart valves is required, but not as the next step. Choice C, cardiac catheterization, would likely occur if an abnormal stress test demonstrating symptoms of myocardial ischemia is found

A 70-year-old man, with a history of pulmonary hypertension and obstructive sleep apnea, presents with complaints of increasing dyspnea while walking his dog. He has also recently noted increased lower extremity edema. On physical examination, jugular venous distension is noted. Auscultation of the chest demonstrates a high-pitched blowing diastolic murmur. The murmur is heard over the second and third left intercostal spaces. An S 3 is appreciated. Abdominal exam reveals hepatomegaly and splenomegaly. Which of the following maneuvers would be the most appropriate to choose for better identification of the murmur? A Left lateral decubitus position listening with the bell of the stethoscope B Standing C Seated leaning forward D Inspiration E Expiration

The Correct Answer is: D Choice D, inspiration, will increase the intensity of the murmur of pulmonic regurgitation/insufficiency. The Valsava maneuver will diminish the intensity of the murmur. Choice B, standing, will cause the mid-systolic click associated with mitral valve prolapsed to move toward S1 or become more audible. Choice A, rolling the patient to the left lateral decubitus position, is most useful in identification of the murmur of mitral stenosis. Choices B, C, and E, have no effect on the Graham Steell murmur of pulmonic regurgitation

During a hospitalization for pneumonia, troponin levels are drawn on a 62-year old-man with a history of hypertension, hyperlipidemia, and chronic tobacco use, and found to be elevated above the 99 th percentile of normal. If acute myocardial infarction is ruled out, which of the following disease entities could also cause troponin elevation? A Mitral regurgitation B Gout C Parkinson's disease D Sepsis E Herpes zoster

The Correct Answer is: D Choice D, sepsis, is one of a long list of disease entities that can cause troponin elevation, including arrhythmias (both tachycardic and bradycardic), aortic valve disease, hypertrophic cardiomyopathy, invasive cardiac surgeries and procedures, severe pulmonary hypertension, pulmonary embolism, myocardial infiltrative diseases (such as amyloidosis, sarcoidosis, scleroderma, and hemochromatosis), acute respiratory failure, burns, pericarditis, endocarditis, myocarditis, and even occasionally due to extreme athletic activities such as marathon running. Not included on this long list, however, are choices A, B, C, and E.

An 8-year-old girl is brought in to the emergency department with abdominal cramps, nausea, and vomiting since early this morning. She has had two loose stools but denies melena or hematochezia. She has had a low-grade fever. In the past hour, her vision has become blurry and she feels increasingly weak. Her mother has had similar but milder symptoms. Twenty-four hour dietary recall includes only chicken broth today. Last night for dinner they had meatloaf (fully cooked), mashed potatoes, and green beans. Her mother cans all their vegetables. Her medical history is unremarkable. She takes no medications. No known drug allergies. Examination reveals a temperature of 99°F, clear lungs, and mildly tachycardic heart with no murmur audible. Abdomen-bowel sounds present, soft with mild diffuse tenderness, no guarding. Neurologic examination is significant for decreased visual acuity and decreased motor strength (2/5) in the upper and lower extremities. The most likely etiology is A enterotoxic E coli B cholera species C pinworms D Clostridium botulinum

The Correct Answer is: D Clostridium botulinum produces a neurotoxin that can lead to life-threatening illness including respiratory paralysis. Botulism infection is caused by the spore-forming bacteria that lives in soil and can be foodborne. In the latter case, home-canned foods are often the cause. After a 12-hour to 3-day incubation period, botulism begins with classic symptoms of abdominal pain, nausea, vomiting, and mild diarrhea and, if unchecked, evolves into a progressive neurologic disorder marked by double vision, motor weakness, and ptosis. Respiratory muscle involvement may occur ultimately and result in death. Because of the virulence of the neurotoxin it has been used as an agent of bioterrorism. Cholera and enterotoxigenic E. coli cause a foodborne diarrheal illness that can result in significant morbidity and mortality, but they do not have neurologic manifestations. Pinworm infection is usually found among younger children, is marked by severe anal itching, and fecal-oral transmission.

A 2-year-old baby girl is brought to the ED with a history of abdominal pain and diarrhea. Mother states that the child was playing normally and then "doubled over" with what appears to be abdominal pain. The abdomen appears slightly distended and is tender to palpation. While in the ED the child has a bloody, diarrheal bowel movement. Which of the following is the most likely diagnosis? A pyloric stenosis B mesenteric ischemia C Crohn disease D intussusception E Hirschsprung disease

The Correct Answer is: D Intussusception is the most frequent cause of intestinal obstruction in the first 2 years of life. The patient develops paroxysms of pain followed by bloody bowel movements. Pyloric stenosis typically presents prior to the age of 6 months with vomiting but not with diarrhea. Hirschsprung disease results from an absence of ganglion cells in the colon and typically presents early in life with failure to pass meconium, followed by vomiting and abdominal distension. The typical age of onset is later in adolescence in Crohn disease and in the elderly in mesenteric ischemia

A 54-year-old man presents to the emergency department with crampy abdominal pain, nausea, and vomiting. The patient has not passed gas or had a bowel movement for at least 10 hours. On examination, the abdomen is distended and there are high-pitched bowel sounds with rushes. A plain radiograph of the abdomen reveals cecal distension to 12 cm. What is the most appropriate definitive management for this patient? A Intravenous fluids B Nasogastric suction C Observation D Surgical exploration

The Correct Answer is: D Massive distention of the cecum, as detected on plain radiograph, is typically seen in "closed loop" obstructions where the ileocecal valve is competent. When distention approaches 12 cm, there is an increased risk of perforation and/or gangrene. Expedient surgical intervention is indicated. Although observation with intravenous fluids and nasogastric decompression are important adjuncts to management, surgical exploration is the only way to rapidly address this emergent situation.

A patient who is intoxicated presents to the emergency department. On ocular exam, you notice mydriasis. Which substance could he have been using? A sedatives B PCP C opioids D cocaine

The Correct Answer is: D Patients intoxicated with cocaine present with mydriasis. In opioid intoxication, the pupils are constricted. PCP intoxication is associated with nystagmus.

A 55-year-old woman with a history of mitral stenosis (secondary to rheumatic heart disease) presents to the emergency department with increasing dyspnea while walking up one flight of stairs. She denies chest pain and discomfort, but states that for last 24 hours she has also noticed palpitations. She also admits to lower extremity edema, which is new within the last week. On EKG, she demonstrates atrial flutter with 2:1 AV block. Which of the following is the most appropriate therapy for this patient? A Aspirin 325 mg B Plavix 75 mg C Plavix 75 mg and aspirin 81 mg D Warfarin, dosed to INRs between 2.0 and 3.0 E Dipyridamole 200 mg and aspirin 25 mg

The Correct Answer is: D Patients with atrial flutter of less than 48 hour duration may be cardioverted without anticoagulation, unless they have mitral valve disease, in which case they should be treated with warfarin; therefore, choice D is the most appropriate choice.

A 56-year-old male who works in construction climbing ladders has developed pain to the right foot for several days. You have seen and examined the patient a few days after the patient started complaining of pain to the foot. Your initial x-rays of the foot are negative for fracture. He continues to have pain, and decreased ability to bear weight. Based on this clinical scenario, how many days after the initial examination should another x-ray be ordered to look for a stress fracture? A 1-2 days B 5-7 days C 10-12 days D 14-30 days E 180 days

The Correct Answer is: D Radiographic evidence of stress fractures is not immediately apparent after the initial onset of symptoms. Estimates of anywhere from 14 to 42 days before visible signs of a stress fracture can be detected on X-ray have been put forth with a commonly utilized range of 14 to 30 days. Any answer choice above that included numbers less than 14 days would simply be inaccurate due to the gradual appearance of the classic x-ray findings of a stress fracture. After 180 days from the time of stress fracture onset, the fracture should be nearly healed if managed properly. If a stress fracture is highly suspected in light of negative x-rays, imaging with a bone scan should help make the diagnosis. MRI scans may help confirm the diagnosis, but they are rarely utilized in the work-up of suspected stress fractures.

A 76-year-old male presents to the hospital with a hot, swollen, and painful right knee that appears to have an effusion. After your examination, you decide to perform an arthrocentesis. The fluid aspirate is turbid, cloudy, and most definitely looks like it is infectious material. Based on this clinical scenario, what would be most likely bacterial organism present in an elderly septic arthritis? A Hemophilus influenzae B Neisseria gonorrhoeae C Pseudomonas aeruginosa D Staphylococcus aureus E Streptococcus pneumonia

The Correct Answer is: D Septic arthritis is an infection within a joint space. The infection can be caused by multiple pathogens including bacteria, viruses, fungi and mycobacteria, but in older adults the most common organism implicated is Staphylococcus aureus. These bacteria can get into the joint space through multiple mechanisms including direct inoculation after a penetrating injury, hematogenous spread from an infection elsewhere in the body, or from extension from a contiguous bone infection. All of the bacteria mentioned as possible answers are possible causes of septic arthritis, but there prevalence varies within different patient populations. Hemophilus influenza and Streptococcus pneumonia are commonly seen in children less than five years of age that develop septic arthritis. Neisseria gonorrhoeae is most common in sexually active adolescents and young adults. Pseudomonas aeruginosa is a common cause of septic arthritis in IV drug abusers.

14-year-old boy presents to the emergency department with acute scrotal pain and vomiting for the past 2 hours. His left testicle is in extreme pain and he states the pain started while playing basketball in gym class. On physical exam of the affected testicle, which of the following findings would suggest testicular torsion? A Transillumination B Positive Prehn's sign C Positive cremasteric reflex D Abnormal transverse lie E Mass of enlarged veins palpated

The Correct Answer is: D Testicular torsion is most common between ages 12-18 with the classic presentation of abrupt and severe onset of pain with nausea/vomiting. The testicle on physical examination is painful, swollen, high-riding, tender, and has an abnormal transverse lie (D). Transillumination (A) is when light is placed behind the scrotum and fluid is illuminated in cases of hydroceles. Prehn's sign (B) is pain relief with elevating the scrotum and is positive in cases of epididymitis. The cremasteric reflex (C) is a normal finding that causes elevation of the testis on the ipsilateral side when the inner aspect of the inner thigh is stroked. The absence of cremasteric reflex on the affected side is often found in acute torsion. A mass of enlarged veins (E), or "bag of worms," is a finding associated with a varicocele.

An 18 month old female is brought to the pediatricians office with a history of cough, fever of 102, and decreased fluid intake. Her immunizations are not up to date as the family just moved to the United States from out of the country. On physical exam she is drooling and sitting up in a "tripod position" with mild stridor. What is the most appropriate initial treatment indicated for this condition? A Humidified air B Albuterol nebulizer C Budesonide nebulizer D Stabilization of airway E Ipratropium nebulizer

The Correct Answer is: D The clinical presentation suggests epiglotitis. This is an emergent airway condition. The anesthesiologist , or the pediatric otolaryngologist must be called to stand by to intubate or insert a tracheostomy if the patients airway closes. Oxygen and antibiotics should administered emergently also. Aerosol steroids are also considered to decrease the edema in the airway. No x-rays are indicated when the presentation is classic. Albuterol is a beta-agonist used for treatment of asthma. Budesonide,a steroid and ipratropium, an anticholinergic agent are most often used in combination with albuterol for treatment emphysema and asthma.

A 28-year-old male presents with hypotension, marked tachypnea, and severe dyspnea following a fistfight. A physical exam reveals ecchymoses over the lateral left rib cage, hyperresonance on the left, and tracheal shift to the right. Which of the following is the most appropriate next step for management of this patient? A Anesthesia consultation B Chest CT scan C Chest x-ray D Needle thoracostomy to the second intercostal space, midclavicular line E Tube thoracostomy to the fifth intercostal space, midclavicular line

The Correct Answer is: D The diagnosis for this patient is a traumatic tension pneumothorax, a true medical emergency. Diagnosis can be made based on physical exam findings. Although definitive treatment with a properly placed tube thoracostomy is preferred, choice E has the position incorrect. Needle decompression in the second intercostal space, midclavicular line is the correct choice, with this remaining in place until a chest tube is properly in place.

You are evaluating a 32-year-old male who has possibly injured his Achilles tendon while planting his foot into the soft ground while playing touch football. When evaluating a patient with this suspected injury, you have the patient plantar flex his ankle as part of the exam. Based on your knowledge of anatomy, what two plantar flexing muscles attach into the Achilles tendon? A Anterior tibialis and soleus B Flexor hallucis longus and flexor digitorum C Flexor hallucis longus and gastrocnemius D Gastrocnemius and soleus E Peroneus longus and peroneus brevis

The Correct Answer is: D The gastrocnemius and soleus muscles form a common tendon at their distal insertions known as the Achilles tendon. This the strongest and thickest tendon in the body and it attaches at its distal end onto the calcaneus bone of the foot. Contraction of the gastrocnemius and soleus muscles are responsible for plantar flexion of the foot and ankle. The anterior tibialis muscle is one of the important dorsiflexors of the foot and ankle. Flexor hallucis longus and flexor digitorum are both involved in plantar flexion, but are not as strong as the gastrocnemius and soleus muscles and do not attach to the Achilles tendon. The flexor hallucis longus is primarily responsible for plantar flexion of the great toe and flexor digitorum produces plantar flexion of the remaining toes. The peroneus longus and peroneus brevis muscles do contribute to plantar flexion, but are not connected to the Achilles tendon. They are also the primary evertor muscles of the foot and ankle

A 21-year-old man presents to the emergency department complaining of a swollen left ear (see image) after he experienced blunt trauma in a collegiate wrestling match. Which of the following is the most likely diagnosis? A Avulsion B Cellulitis C Contusion D Hematoma E Cauliflower ear

The Correct Answer is: D The history of blunt trauma resulting in acute swelling, ecchymosis, and loss of cartilaginous landmarks is consistent with auricular hematoma (D). An auricular contusion (C) would result in maintenance of the landmarks. The patient currently lacks cauliflower ear (E), but is at risk of developing this deformity as a result of this hematoma.

A 23-year-old man, unrestrained driver, is brought to the emergency department by ambulance after having been involved in an automobile accident. His vitals are BP: 99/54 mm Hg, P: 112/min, R: 18/min, oxygen saturation: 99%, T: 99.8°F. Examination reveals mild abdominal tenderness with pain radiating to the right shoulder. What is the most appropriate diagnostic test to order initially? A Computed tomography of the abdomen and pelvis B Diagnostic peritoneal lavage C Flat and upright abdominal radiographs D Diagnostic ultrasound

The Correct Answer is: D The initial evaluation of blunt abdominal trauma is by the performance of a FAST (focused assessment with sonography for trauma) ultrasound, which is performed by an emergency department physician or surgeon. CT scan remains an adjunct test in hemodynamically stable patients or in patients in whom further assessment of solid intra-abdominal organs is required.

23-year-old man presents to the outpatient clinic for follow-up from a recent urgent care visit. He complains of sore throat, fever, fatigue, myalgias, and a rash that started 5 days ago, and have worsened since he was seen in the urgent care 3 days ago. The patient appears non-toxic with a temperature of 39.4 degrees Celsius. Physical exam reveals pharyngeal and tonsillar erythema without exudates, generalized lymphadenopathy, a morbilliform rash on his trunk, and no hepatosplenomegaly. A rapid strep screen and Monospot performed at the local urgent care were reportedly negative. Which of the following laboratory tests is most likely to confirm the expected diagnosis? A Complete blood count B Cytomegalovirus titer C Group A beta-hemolytic strep culture D HIV viral load E Epstein-Barr virus titer

The Correct Answer is: D The patient presentation is consistent with acute retroviral syndrome, which is best confirmed during this initial presentation phase through direct testing for the HIV virus, such as an HIV viral load (D). The lack of tonsillar exudates, a negative monospot, and presence of generalized adenopathy make infectious mononucleosis (B and E) less likely. A CBC (A) may show lymphopenia and support the diagnosis, but it doesn't confirm the diagnosis.

A 67-year-old female with a history of oxygen dependent emphysema presents with a 4-hour history of increasing shortness of breath and pleuritic chest pain on the right side. Her resting oxygen saturation rate is 90%, and she is having pain on inspiration. On examination, the patient has decreased lung sounds with wheeze on the left and absent sounds on the right. There is also tympany to percussion on the right. Based on these findings, what is the best therapy for this patient? A Needle insertion to right chest wall B Supportive care C Increased oxygen delivery D Chest tube insertion E Nebulized albuterol

The Correct Answer is: D The treatment for this patient, who has a pneumothorax, is chest tube insertion and reinflation of the lung. Once the air leak has been eliminated and the lung appears reinflated on serial chest x-rays, the chest tube may be removed.

A 22-year-old African American male presents to the emergency department with shortness of breath, which started 2 hours prior to arrival. He does not have a history of pulmonary disease that he is aware of, and he states that in the past at random events he has had similar episodes. He does nothing to get the episodes to stop, and he also states that he feels his chest pounding at the same time of the shortness of breath. He has no medical history that he is aware of, and he takes no medications or any illicit drugs. On examination he is alert, awake, and oriented. His vital signs show T 99.0, P 142, R 18, and BP 132/82. His chest x-ray is negative for any acute cardiopulmonary disease, and his electrocardiogram has an irregularly irregular rate of 142 with visible delta waves. Given the clinical situation above, what is the best medication for managing this patient's condition with a long-term approach? A Amiodarone B Atropine C Hydralazine D Flecainide E Digoxin

The Correct Answer is: D This patient has Wolff-Parkinson-White (WPW) syndrome along with atrial fibrillation and a rapid ventricular response. Of the choices given in managing this patient's tachycardia, oral flecainide (D) will serve to slow the process within the accessory pathway and prolong the refractory period. Amiodarone (A) has been shown to not be effective in managing the patient's tachycardia with respect to WPW, and the other medications (B, C, and E) would not have an effect on the condition and may actually worsen the patient's symptoms.

A 24-year-old male is brought to the emergency department by his girlfriend. She states that he began having a seizure in the car approximately seven minutes ago. She is not sure of his medications, but states he has a known seizure history and has seizures a few times a year. The patient is currently actively seizing. Which of the following is the first-line agent to give this patient? A Carbamazepine B Gabapentin C Levetiracetam D Lorazepam E Phenytoin

The Correct Answer is: D This patient has a known history of seizures, with current ongoing seizure activity and the concern of status epilepticus, a true emergency. Prolonged seizure activity is associated with hyperthermia, metabolic disturbances, cardiopulmonary dysfunction, and irreversible damage.

A 42-year-old healthy male presents to the emergency department with the complaint of a progressively worsening sore throat, and difficulty swallowing over the past 48 hours. He also complains of a subjective fever, but denies any headaches, nausea, or vomiting. On exam, the patient is afebrile and in mild distress, with a presentation of leaning forward on the exam table. His TM examination is normal, there is no rhinorrhea, and the oropharynx is patent without signs of stridor. His lungs are clear, and he has a regular rhythm on cardiac exam. What diagnostic test is indicated for a definitive diagnosis? A Chest x-ray B Complete blood count (CBC) C Nasal bacterial culture D Soft tissue neck x-ray E CT scan of the neck

The Correct Answer is: D This patient has a presentation that is consistent with acute epiglottitis. While ensuring that the airway is patent and the patient can maintain the airway, the first step in determining the diagnosis is a soft tissue neck x-ray, to determine inflammation to the epiglottis. While rare, epiglottitis can be from a bacterial infection, and can be quite serious and sometimes fatal.

A 70-year-old man with a history of pulmonary hypertension and obstructive sleep apnea presents with complaints of increasing dyspnea while walking his dog. He has also recently noted increased lower extremity edema. On physical examination, jugular venous distension is noted. Auscultation of the chest demonstrates a high-pitched blowing diastolic murmur. With inspiration, the murmur increases in intensity and is heard over the second and third left intercostal spaces. An S 3 is appreciated. Palpation of the precordium reveals a hyperdynamic right ventricle, and both a systolic and diastolic thrill. The abdominal exam reveals hepatomegaly and splenomegaly. Based on this patient's history and physical exam findings, which of the following is the most likely finding on echocardiogram? A Tricuspid regurgitation B Aortic stenosis C Atrial septal defect D Pulmonic regurgitation/insufficiency E Mitral stenosis

The Correct Answer is: D This patient is demonstrating signs and symptoms of right heart failure, and with a history of pulmonary hypertension and a high-pitched diastolic blowing murmur (Graham Steell murmur), Choice D is the most likely of the choices offered. A blowing holosystolic murmur at the left lower sternal border is characteristic for tricuspid regurgitation, Choice A. Choice B, aortic stenosis, presents with a systolic ejection murmur. An atrial septal defect, Choice C, if large, could present with similar symptoms of exertional dyspnea secondary to a large shunt, but auscultation would reveal a moderately loud systolic ejection murmur that is heard best in the second and third interspaces. This is secondary to increased pulmonary arterial flow. Choice E, mitral stenosis, presents with a diastolic murmur heard best in the left lateral decubitus position, with the bell of the stethoscope at the apex.

A 22-year-old African American male presents to the emergency department with shortness of breath, which started 2 hours prior to arrival. He does not have a history of pulmonary disease that he is aware of, and he states that in the past at random events he has had similar episodes. He does nothing to get the episodes to stop, and he also states that he feels his chest pounding at the same time of the shortness of breath. He has no medical history that he is aware of, and he takes no medications or any illicit drugs. On examination he is alert, awake, and oriented. His vital signs show T 99.0, P 142, R 18, and BP 132/82. His chest x-ray is negative for any acute cardiopulmonary disease, and his electrocardiogram has an irregularly irregular rate of 142 with visible delta waves. Based on the information provided, what anatomical pathway is responsible for the above findings of the delta waves? A Intranodal pathway B Bachmann's bundle C Perkinje fibers D Bundle of Kent E Bundle of His

The Correct Answer is: D This patient is exhibiting Wolff-Parkinson-White (WPW) syndrome by evidence of the delta waves on the electrocardiogram. The anatomic pathway that is known to be the causative abnormality in WPW is the Bundle of Kent (D). Intranodal pathway (A), Bachmann's bundle (B), Perkinje (C), and His bundles (E) are not the anatomic pathways found with WPW.

A 44-year-old female presents to the emergency department with a right-sided headache. She states the headache is located on the right temple region, is non-radiating, and does not cause photophobia. She is otherwise healthy and has no reported medical problems. She only takes acetaminophen for the pain, which has minimal relief. On physical exam she is alert, awake, and oriented. Her vitals are T 98.8, P 78, R 18, and BP 128/76. Her head is normocephalic, atraumatic, and pupils are equal and reactive. She has tenderness to the right temporal area of the temporal artery, and there is no noted swelling, redness, or abnormalities noted. There is no tenderness to the cervical muscles. She exhibits a non-focal neurological exam. Based on the description, what is the most likely diagnosis of this patient? A Migraine headache B Tension headache C Cluster headache D Temporal arteritis E Takayasu arteritis

The Correct Answer is: D This patient is exhibiting a case of temporal arteritis (D). She has the signs and symptoms that are classic in nature and do not fit into the realm of the other headaches (A, B, and C), all of which usually have different characteristics on history and physical exam. Takayasu arteritis (E) will typically not present in the temporal artery.

A 66-year-old male with a history of hypertension, diabetes mellitus, and hypercholesterolemia presents by emergency medical services (EMS) to the emergency department complaining of severe chest pain with radiation into his back. The patient states that he was feeling well in the morning, but while performing some light activity he felt a "ripping" sensation in his back, which he initially thought was a pulled muscle. The pain continued and the patient started to have chest pain, shortness of breath, and lightheadedness. On initial examination the patient is still in pain, pale, diaphoretic, and has a blood pressure of 85/40. His chest is clear to auscultation, and he has a 3/6 diastolic murmur best appreciated at the base of the heart. Given this clinical scenario, what would be the expected finding on chest x-ray? A Normal B Pleural effusion C Decreased lung volume D Widened mediastinum E Cardiomegaly

The Correct Answer is: D This patient is exhibiting a history and physical examination that is consistent with a thoracic aneurysm. The patient's history of hypertension, along with the "ripping" sensation in his back and hypotension give a clinical presentation that is suggestive of a thoracic aneurysm dissection. In this instance it would be expected that the patient would have evidence of a widened mediastinum (D) on chest radiography. While pleural effusions (B), decreased lung volumes (C), and cardiomegaly (E) can all be present on a chest film with an aneurysm, the widened mediastinum would be the expected finding if you suspect an aneurysm. It is very unlikely that a thoracic aneurysm with hypotension would have a normal chest radiograph finding (A).

A 29-year-old female who is only taking oral contraceptives presents to the emergency department with a 1-day history of worsening shortness of breath. On examination, the patient is afebrile, her pulse rate is 105, respiratory rate is 24, and blood pressure is 122/78. She has wheezing to all lung fields and appears to be in mild to moderate distress. There is no swelling or edema to the lower extremities. Based on these findings, what is the best test to order to determine the diagnosis in this patient? A CT chest B Chest x-ray C Ventilation to perfusion scan D Pulmonary arteriography E MRI chest

The Correct Answer is: D This patient presents with a history and physical exam that is consistent with an acute pulmonary embolus (PE). The prognosis for this type of illness can be serious, and in some cases death can result. The test that is still the gold standard for determination of a PE is the pulmonary arteriogram. CT of the chest with contrast tends to be the more utilized testing modality, but the arteriogram remains the test of choice.

A 55-year-old right-hand dominant man presents with a 4-hour history of weakness and tingling of his right hand and numbness of the right side of his mouth. Mild difficulty was noted with word finding. His symptoms have improved since onset but have not fully resolved. There is no significant medical history. Physical examination revealed flat right nasolabial fold, subjective numbness of the right hand, right pronator drift, clumsiness of finger tapping on the right hand, increased deep tendon reflexes on the right, as well as a present Babinski. What is the most likely etiology for this patient's problem? A migraine headache B peripheral neuropathy C syncope D transient ischemic attack E seizure

The Correct Answer is: D Three key features of a transient ischemic attack include sudden onset and complete reversal of symptoms within 24 hours, usually within 15 minutes. The symptoms are usually in the anatomical distribution of a single blood vessel. This patient's history is not suggestive of migraine or syncope. His physical examination findings do not correlate with peripheral neuropathy or seizure.

A 88-year-old female is found lying on the ground in a supine position after sustaining a fall in her house from tripping on the rug.. Her leg is shortened, abducted and externally rotated, and she is complaining of pain to the right leg with radiation to the knee. Based on the history of the patient, what type of injury has this patient sustained? A Non-displaced pelvic fracture B Hip dislocation C Femoral shaft fracture D Femoral neck fracture E Distal femur fracture

The Correct Answer is: D When a fracture occurs in the femoral neck that includes displacement, the leg appears shortened, abducted and externally rotated. The tension that exists from muscles attached above and below the fracture site results in shortening of leg length after the fracture as the muscles contract. If there is no displacement of the bone, no leg length changes should occur. Generally, hip fractures would not result in a lengthening of the leg, so all answers that include this option would be wrong. When the leg appears shortened, flexed, adducted, and internally rotated, a posterior hip dislocation should be suspected. A leg that is flexed, abducted and externally rotated suggests an anterior hip dislocation. Posterior hip dislocations are far more common than anterior dislocations, accounting for more than 90% of all hip dislocations.

9-year-old female presents to the emergency department after falling out of a second story window. Imaging of the left upper extremity shows the following fracture pattern: How would this fracture be described? A Transverse B Oblique C Spiral D Segmental E Greenstick

The Correct Answer is: E A greenstick fracture occurs almost exclusively in young children as a result of their bones being more pliable than those of adults. On imaging, a greenstick fracture has cortical disruption on the convex side of the bone with an intact periosteum on the concave side of the fracture. Spiral fractures (C) are often seen in non-accidental or child abuse cases. Transverse (A), segmental (D), and oblique (B) fractures involve both cortices.

Which of the following indicates EKG changes consistent with a suspected pulmonary embolus? A Increased QRS amplitude with tall R waves in limb leads and deep S waves in V1 and V2 B Notched P wave in leads I and II with an increased duration C Prolonged PR interval with peaked T waves D Prolonged QRS duration with QS complex in V1, monophasic R wave in leads I and V6 E Sinus tachycardia with a right ventricular strain pattern, prominent S in lead I, Q wave and inverted T in lead III

The Correct Answer is: E Approximately 70% of patients with a pulmonary embolus will demonstrate EKG changes. The changes present may range from sinus tachycardia to findings consistent with marked right heart strain and dilatation. The correct answer also includes the classic "S1Q3T3" pattern of changes associated with pulmonary embolism. The findings in choice A are consistent with left ventricular hypertrophy. Choice B is indicative of left atrial enlargement. Choice C is consistent with hyperkalemia. Choice D indicates left bundle branch block. These changes are not commonly associated with pulmonary embolism.

A fracture involving the medial epicondyle will most likely cause damage to which nerve? A Axillary B Median C Peroneal D Radial E Ulnar

The Correct Answer is: E Because the ulnar nerve passes through the cubital tunnel, which is a groove on the posterior aspect of the medial epicondyle, any fractures involving the medial epicondyle can also cause damage to the ulnar nerve. The median nerve is most susceptible to injury at the carpal tunnel. Branches of the radial nerve can become entrapped on the lateral side of the elbow and the associated symptoms are often confused with lateral epicondylitis. Radial nerve injuries are more commonly associated with humeral shaft fractures. The axillary nerve is significantly proximal to the medial epicondyle and the peroneal nerve is in the leg.

A 65-year-old man presents to the emergency department with an acute ischemic stroke. His CT scan is normal. His blood pressure is 180/100 mm Hg. What is the most appropriate treatment for his hypertension? A labetalol (Normodyne) 20 mg IV B nifedipine (Procardia) 10 mg po C nitroprusside (Nipride) drip at 1 mg/kg/min D clonidine (Catapres) 0.1 mg po E no antihypertensive at this time

The Correct Answer is: E Blood pressure is typically elevated at the time of presentation in acute ischemic stroke. It will decline without medication in the first few hours to days. Aggressively lowering blood pressure in an acute ischemic stroke may decrease the blood flow to the ischemic but salvageable brain tissue. This potentially salvageable brain tissue is referred to as the penumbra. Decreasing blood flow to the ischemic penumbra by acutely lowering blood pressure may result in eventual infarction of this brain tissue. Treatment of previously undiagnosed hypertension should be deferred for several days. Blood pressure should be treated if there are other indications, such as angina or heart failure. Control of blood pressure is appropriate in patients who are receiving tissue plasminogen activator (t-Pa) for their stroke. Blood pressure should be lowered cautiously to a systolic of less than 185 mm Hg and a diastolic of less than 110 mm Hg.

A 33-year-old IV drug user presents to the emergency department with chills, diaphoresis, anorexia, and malaise. On physical exam, her temperature is 40C, BP 98/55, P 115 bpm, and RR 22. Two separate blood cultures are positive for S.aureus. Which of the following diagnostic studies would be most useful in establishing this patient's diagnosis? A EKG B CXR C Rheumatoid factor D ESR E TEE

The Correct Answer is: E Choice E, TEE (or transesophageal echocardiogram), would be most useful in establishing a diagnosis of infective endocarditis, as a positive echocardiogram demonstrating presence of a vegetation would satisfy one of the Duke criteria's major criteria. TEE is more sensitive than TTE (transthoracic echocardiogram) for detecting vegetations. EKG and CXR should be performed as part of this patient's evaluation, but would be less useful than TEE in establishing a diagnosis of infective endocarditis. ESR and rheumatoid factor are frequently elevated in patients with endocarditis, but are not specific to the diagnosis of endocarditis.

65-year-old recent alcoholic comes to the emergency department with recent onset of dyspnea, with exertion, 3 pillow orthopnea, lower extremity edema, and palpitations, in which he describes his heart as racing. Which of the following is most likely to be the cause of his high-output congestive heart failure? A Mitral regurgitation B Aortic stenosis C Uncontrolled hypertension D Ruptured chordae tendinae E Beriberi

The Correct Answer is: E Choice E, beriberi, also known as thiamine deficiency, is common among alcoholics, and the only high-output cause of congestive heart failure among the choices offered. Other causes include severe anemia, thyrotoxicosis, and arteriovenouis shunting (for example, in hemodialysis patients). Choice A, mitral regurgitation, is a cause of excessive preload, leading to heart failure. Choice D, ruptured chordate tendinae associated with mitral regurgitation, would also be a cause of excessive preload, leading to heart failure. Choices B and C, aortic stenosis and uncontrolled hypertension, are causes in which too much afterload leads to heart failure.

24-year-old man with a recent history of a viral illness comes to the emergency department complaining of severe left-sided chest discomfort, which radiates through to the left trapezius region. On coming into the room, you note that he is sitting up and hunched forward. On physical examination, the patient's temperature is 39°C, blood pressure is 135/78, with a pulse of 85 bpm, and a pericardial friction rub is noted. Laboratory findings demonstrate elevated serum creatine kinase levels and normal serial troponin levels. His EKG demonstrates peaked T waves. His CXR demonstrates no acute process. Which of the following is the most appropriate treatment for this patient? A Morphine B Enoxaparin C Nitroglycerin D Penicillin V E Indomethacin

The Correct Answer is: E Choice E, indomethacin 25-75 mg QID, and bed rest would be the most appropriate treatment in a patient with acute viral pericarditis, as a nonsteroidal anti-inflammatory agent will ameliorate the inflammatory process. Choices A and C are appropriate in a patient suspected of acute coronary syndrome. Choice B, enoxaparin, is contraindicated in patients with pericarditis, as anticoagulants could lead to worsening of pericardial effusion and cardiac tamponade, especially if it is secondary to bleeding into the pericardial space, such as with trauma or postoperatively

50-year-old woman with a history of hypertension complains of chest tightness and dyspnea while walking up one flight of stairs. She recently experienced an episode of near-syncope while walking her dog. She denies a history of rheumatic fever. On auscultation, a crescendo-decrescendo systolic ejection murmur is heard at the upper right sternal border, radiating to the carotids bilaterally. Troponin levels are negative at 0, 3, and 6 hours. Her EKG demonstrates evidence of left ventricular hypertrophy. Given the patient's physical exam findings and recent symptoms, which of the following is the most appropriate next diagnostic study? A Chest X-ray B Transesophageal echocardiogram C Holter monitor D Treadmill exercise stress test E Transthoracic echocardiogram

The Correct Answer is: E Choice E, transthoracic echocardiogram, is a simple, sensitive, and non-invasive diagnostic tool which can evaluate for the presence of valvulopathy in a patient in this age group, who is likely demonstrating severe aortic stenosis secondary to a congenital bicuspid valve. Patients with a congenital bicuspid aortic valve typically develop symptoms once the valve leaflets have become calcified and thickened, secondary to the undue stress over many years on a structurally abnormal aortic valve. Choice A might be able to give evidence of cardiomegaly or calcification of heart valves, but would not be sensitive enough to detect the degree of valvulopathy, if present. Choice B, transesophageal echocardiogram, would give information regarding valvulopathy, but is a more invasive test; therefore, choice E is more appropriate. Choice C is a useful diagnostic tool for evaluation of patients complaining of palpitations, but incorrect for this patient, who has no symptoms of palpitations. Choice D, although a useful diagnostic tool for the evaluation of exercise tolerance and in patients complaining of chest pain, does not allow direct visualization of the heart valves to evaluate the degree of aortic stenosis; as the patient is likely demonstrating severe aortic stenosis, cardiac catheterization to evaluate for coronary artery disease prior to surgery will need to be performed

Which of the following is the most reliable clinical tool for confirming endotracheal intubation in an emergency situation? A Auscultation over the stomach B Endotracheal tube condensation C Pulse oximetry monitoring D Sellick maneuver E Visualizing the tube passing through the vocal cords

The Correct Answer is: E Clinical assessments and practices used to assess tube placement, and help with placement, such as auscultating for breath sounds and noise within the stomach, have not had a confirmation rate comparable to directly visualizing the tube passing through the vocal cords. Tube condensation may occur with esophageal intubation as well. The Sellick maneuver may help with correct positioning, but is not a confirmatory test. Once placement is suspected, confirmation with an end-tidal CO 2 detector and chest x-ray is recommended. Pulse oximetry measurement should be performed throughout the intubation, with decreased saturations representing a worsening clinical condition and/or esophageal intubation.

What is the most common radiologic finding in a patient with a tension pneumothorax? A Pleural effusion B Infiltrate C Enlarged cardiac silhouette D Elevated diaphragm E Mediastinal shift

The Correct Answer is: E Collapse of the lung, followed by an increase in intrapleural pressure, will lead to a tension pneumothorax. In most cases, the air leaks through the subcutaneous spaces. But in a tension pneumothorax, the air is locked in and creates a life-threatening emergency

A 46-year-old female with a history of poorly-controlled diabetes and grand mal seizures presents for evaluation of bilateral foot pain. She describes the pain as burning and has noticed it is worse at night. She occasionally has this pain in her feet. She denies other medical conditions, and her medications are metformin and dilantin. Which of the following medications, used for the suspected diagnosis, should be avoided in this patient? A Capsaicin B Desipramine C Gabapentin D Lidocaine patch E Tramadol

The Correct Answer is: E Diabetic peripheral neuropathy is the diagnosis, and describes any neuropathy in the diabetic patient. This patient is exhibiting a distal symmetric polyneuropathy with the classic associated symptoms, commonly called "pins and needles" by patients. When associated with pain and functional impact, pharmacologic therapy is warranted. There are many agents to choose from, with each of the answer choices being options. However, in a patient with a known seizure disorder, tramadol should be avoided, as it decreases the seizure threshold. Gabapentin, also a seizure medication, may be used, but close monitoring is suggested.

Approximately what percentage of patients with a solid primary tumor elsewhere will end up with metastatic disease of the vertebrae during the clinical course of their cancer? A 10% B 20% C 30% D 40% E 50%

The Correct Answer is: E Fifty percent of cancer patients will develop metastatic disease of the vertebrae at some point during the course of their illness. The highest percentages of cancers that lead to such spinal lesions are carcinomas of the breast, lung, prostate, colon, thyroid and kidney.

A 64-year-old female who has a history of injectable drug use presents with blood work that reveals leukocytosis with a left shift, and there is suspicion of osteomyelitis based on the patient's prior history. Based on this history, what bone would be most affected by hematogenous osteomyelitis in adults? A Feet B Long bones C Pelvis D Sternoclavicular bones E Vertebrae

The Correct Answer is: E Hematogenous osteomyelitis accounts for about 20% of all cases of osteomyelitis in adults. It is more common in males and the prevalence is higher amongst those who are IV drug abusers, patients being treated with dialysis or who have sickle cell disease. Other conditions which may lead to sepsis (i.e. patients with central lines, urinary infections, and urethral catheterization) increase the risk of hematogenous osteomyelitis. Unlike children, the long bones are rarely affected in adults with the vertebrae being the most likely location for the bone infection to occur. Lumbar vertebrae are most often affected, followed by thoracic and cervical vertebrae. Osteomyelitis of the sternoclavicular bones and pelvic bones are not uncommon sites, but these tend to be most frequent amongst IV drug abusers. The feet do not tend to be significantly affected by hematogenous osteomyelitis as frequently as osteomyelitis caused by infected foot ulcers as are often seen in those with diabetes or peripheral vascular disease.

An 18 month infant with congenital heart disease is diagnosed with acute bronchiolitis secondary to respiratory syncytial virus. Which of the following therapies should be initiated? A Albuterol B Amoxicillin C Azithromycin D Prednisone E Ribavirin

The Correct Answer is: E High-risk infants (i.e., congenital heart disease) who develop RSV are eligible for treatment with ribavirin. Antibiotics (B, C) are indicated if secondary bacterial infections develop. Albuterol (A) and prednisone (D) haven't been shown to improve RSV-related bronchiolitis.

A 56-year-old insulin dependent diabetic has been under your evaluation for his diabetes for several years. The patient has a 3-year history of diabetic neuropathy to the right foot, and may have suffered an injury to the foot without knowing due to loss of sensation. The patient now presents with a tender, reddened, and swollen right foot for the last 10 days that is also warm to the touch. You suspect that this patient may have an acute case of osteomyelitis. Based on this history, what bacterial organism is most commonly the cause of osteomyelitis? A Group A beta-hemolytic streptococci B Hemophilus influenzae C Mycoplasma D Pseudomonas aeruginosa E Staphylococcus aureus

The Correct Answer is: E Osteomyelitis is an infection in a bone and can occur in patients of all ages. The most common organism implicated in osteomyelitis across all age groups is Staphylococcus aureus. It can enter the bone through multiple mechanisms including by direct inoculation during an open fracture or during surgical intervention following a fracture (most common mechanisms for adults) or by hematogenous spread from another source (the usual cause in children). Hemophilus influenzae was a much more common organism in pediatric osteomyelitis in the past, but its prevalence is decreasing due to routine immunizations. Group A beta-hemolytic streptococci is the second most common organism found in osteomyelitis in children, while Pseudomonas aeruginosa is the second most common pathogen in adults. Mycoplasma induced osteomyelitis is relatively rare and usually confined to immunocompromised patients.

A 32-year-old man who is HIV positive has a seizure. On presentation to the emergency department (ED) he is confused and unsure of what happened. His partner reports that he had been complaining of headache in the days preceding the event. CT scanning of the head demonstrates five peripheral contrast-enhancing lesions. What is the most likely diagnosis? A AIDS dementia complex B central nervous system lymphoma C cryptococcal meningitis D progressive multifocal leukoencephalopathy E toxoplasmosis

The Correct Answer is: E The most common space-occupying CNS lesion in patients with HIV is toxoplasmosis. This condition may present with headache, focal neurologic deficits, seizures, and/or mental status changes. The typical appearance on brain imaging is that of multiple contrast-enhancing lesions in the periphery, particularly the basal ganglia. CNS lymphoma is more typically a single lesion. AIDS dementia complex presents a diagnosis of exclusion, without a characteristic appearance on imaging. The diagnosis of cryptococcal meningitis is made by examination of the spinal fluid, while PML imaging shows nonenhancing white matter lesions without mass effect.

A 36 year-old woman with no significant past medical history presents with gradual onset of dyspnea and fatigue leading to an episode of "fainting" this morning. Physical exam reveals increased jugular venous pressure, weak carotid pulses, clear lungs, and a loud S2. What is the most likely diagnosis? A Aortic stenosis B Cardiac tamponade C Mitral Regurgitation D Pulmonary fibrosis E Pulmonary hypertension

The Correct Answer is: E The patients symptoms are due to decreased cardiac output resulting from decreased preload associated with pulmonary hypertension (E). Aortic stenosis (A) presents more commonly in geriatric patients who present with a murmur. Cardiac tamponade (B) can decrease cardiac output, but would lead to decreased heart sounds. Mitral regurgitation (C) would cause pulmonary edema and rales in conjunction with increased jugular venous pressure. Pulmonary fibrosis (E) is unlikely in this patient with normal lung sounds.

You are evaluating a patient who is having decreased sensation to his arm after he sustained a head on injury while playing football. He is stating that his right arm has decreased sensation that goes into the hand, but he is slightly vague on being descriptive. As you perform the physical examination, what part of the body would exhibit sensation for the C7 component of the brachial plexus? A Lateral forearm B Lateral upper arm C Medial forearm D Medial upper arm E Third Finger

The Correct Answer is: E The sensory nerve emanating from the C7 level is responsible for sensation of the third finger. The lateral forearm is supplied by C6. The lateral upper arm is supplied by C5. The medial upper arm is supplied T1 and the medial forearm is supplied by C8.

A 46-year-old male is evaluated for a fall from approximately 15 feet, landing on his legs. There is intense pain and swelling to the lower extremity, with decreased sensation. What bone is fractured and is most commonly associated with acute compartment syndrome? A Femur B Patella C Navicular D Talus E Tibia

The Correct Answer is: E The tibia is the most common bone fracture that leads to acute compartment syndrome. It can also occur after other long bone fractures of the arms and legs. While it does occur in the hands and feet, it is unlikely that the fracture of a small carpal bone (scaphoid) or tarsal bone (talus) would result in compartment syndrome. Damage to the hyoid bone in the neck and the patella in the knee are not associated with compartment syndrome

The most commonly fractured long bone in both adults and children is which of the following? A Femur B Fibula C Humerus D Radius E Tibia

The Correct Answer is: E The tibia is the most commonly fractured long bone in the body for both adults and children. The fractures are often the result of sporting activities in the young and may occur from a simple fall in the elderly - especially those with osteoporosis. Motor vehicle accidents are another common cause of tibial fractures. Open or complex tibial fractures are sometimes associated with compartment syndromes, infection and neurovascular compromise. The femur is the strongest of the long bones and generally only sustains fractures when exposed to extreme stress, such as that experienced in a motor vehicle collision or industrial accident. Fibular fractures commonly occur with a direct below to the lateral lower leg or with extreme ankle rotational forces or excessive inversion. A high percentage of ankle fractures involve the fibula, especially in older women. Humerus fractures are relatively rare in adults, but are the second most common fractures to occur at birth - behind only the clavicle in frequency. The radius is the most commonly fractured bone in the upper extremity, but still less common in frequency than the tibia. Falls on an outstretched arm are a common mechanism for the injury

Active elbow extension is primarily controlled by which muscle(s)? A Anconeus B Biceps C Brachialis D Brachioradialis E Triceps

The Correct Answer is: E The triceps are the primary muscles that produce active elbow extension. The anconeus muscle is known as a secondary elbow extensor. The biceps and brachialis muscles are primary muscles of elbow flexion and the brachioradialis is a secondary elbow flexor.

76-year-old man with a history of HTN and diabetes mellitus, type 2, presents to the emergency department with complaints of palpitations, tachypnea, and chest pain. He denies history of CAD, stroke, TIA, or congestive heart failure. He is afebrile, with vital signs as follows: BP 145/98, HR 138, and RR 22. His EKG is shown (Figure 1). Troponins are negative X 3. His echocardiogram demonstrates normal LV systolic function and normal valvular function. Which of the following questions, when answered, will help to determine the next course of therapy? A How long has the patient been a diabetic? B Has the patient ever had an allergic reaction to aspirin? C Does the patient have a family history of dysrhythmia? D Is the patient a smoker? E How long has the patient been experiencing palpitations?

The Correct Answer is: E This patient has atrial fibrillation. Choice E is the most important question to ask to determine the next course of therapy. If the patient has been experiencing definitive symptoms for less than 48 hours, direct current cardioversion can be performed, as the risk of atrial clot formation and thromboembolism is low. Choice A is incorrect because the CHADS2 scoring system does not take into account how long the patient has been diabetic. Choice B is incorrect because aspirin therapy is not utilized. Choice C is incorrect because family history is not a factor in determining the patient's treatment plan. Choice D is incorrect because smoking status is used in calculating Framingham risk score, not a CHADS2 score.

A 1-year-old boy is brought to the emergency department by his parents, who state that the child refuses to walk or crawl and begins crying when they stand him. He seems calm while lying on the examination table. Vitals are as follows: Temp: 38°C, HR: 70, RR: 15. Bruising is noted in several places. His parents deny trauma, but have noticed that he bruises easily. What other physical finding would you expect? A Conjunctival hemorrhages secondary to shaken baby syndrome B Pain response over the wrists secondary to passive range of motion C Pain response with passive range of motion to the hip secondary to slipped epiphysis D Pharyngitis and sand paper rash secondary to a staph infection E Swelling and warmth over the knee secondary to hemarthroses

The Correct Answer is: E This patient has hemophilia A. Hemarthroses usually occur when an affected child begins to walk. Due to his hemophilia, easy bruising can occur. Hemarthroses can cause low-grade fevers without infection being present, so choice D is incorrect. Wrist joints are less involved then knees, ankles, and elbows.

66-year-old male with a history of hypertension, diabetes mellitus, and hypercholesterolemia presents by emergency medical services (EMS) to the emergency department complaining of severe chest pain with radiation into his back. The patient states that he was feeling well in the morning, but while performing some light activity he felt a "ripping" sensation in his back, which he initially thought was a pulled muscle. The pain continued and the patient started to have chest pain, shortness of breath, and lightheadedness. On initial examination the patient is still in pain, pale, diaphoretic, and has a blood pressure of 85/40. His chest is clear to auscultation, and he has a 3/6 diastolic murmur best appreciated at the base of the heart. Given this clinical scenario, what is the best test to definitively diagnose this medical problem? A Chest x-ray B Transthoracic echocardiography C Transesophageal echocardiography D Cardiac catheterization E Computed tomography

The Correct Answer is: E This patient is exhibiting a history and physical examination that is consistent with a thoracic aneurysm. The patient's history of hypertension, along with the "ripping" sensation in his back and hypotension give a clinical presentation that is suggestive of a thoracic aneurysm dissection. Given this clinical situation, the best test to evaluate for a potential dissection is by computed tomography (E). This test is sensitive enough to determine if there is a luminal irregularity. While echocardiography (B and C) may be able to show evidence of an aneurysm, it is not specific enough to show all the areas of an aneurysm. In this situation a chest x-ray (A) would not give enough specificity to appropriately diagnose an aneurysm, as well as a cardiac catheterization (D).

A 68-year-old male presents with the complaint of palpitations in the center of his chest over the last few hours. The symptoms come and go, and last anywhere from 30 seconds to a few minutes. There is some associated lightheadedness with these episodes, and occasionally there is mild shortness of breath. While examining the patient, he has another episode. During this time, the patient's vital signs reveal a pulse of 170 and a blood pressure of 118/69. Based on this history and the findings on the exam and rhythm strip shown, what is the best treatment for this patient? A Diltiazem B Digoxin C Lisinopril D Metoprolol E Amiodarone

The Correct Answer is: E This patient is having non-sustained ventricular tachycardia (NSVT). Based on the clinical presentation, this patient can benefit from an infusion of amiodarone. The dose is normally a 150mg bolus, followed by a drip of 1mg/min for the first 6 hours, then followed by 0.5mg/min for the next 18 to 24 hours. The other agents would not be able to maintain a rhythm for this type of abnormality.

An adult male, not previously vaccinated for rabies, presents to the emergency department after being bitten by an aggressive stray dog. The dog was captured, and declared "probably rabid" by a local veterinarian. Which of the following treatment options should you select for this patient? A administer human rabies immune globulin only B administer equine rabies antiserum only C administer human rabies immune globulin and equine rabies antiserum D administer human rabies immune globulin and equine rabies antiserum and human diploid cell rabies vaccine E administer human rabies immune globulin and human diploid cell rabies vaccine

The Correct Answer is: E Transmission of rabies to this patient must be seriously considered, and postexposure immunization should begin immediately by the administration of human rabies immune globulin (HRIG; 20 IU/kg). About half the HRIG should be infiltrated around the bite wound, and the remainder injected intramuscularly. Human diploid cell rabies vaccine (HDCV) should also be given (1 mL IM in the deltoid), and again on days 3, 7, 14, and 28. HDCV should be delivered in a different syringe and administered at a different site than HRIG

A 77-year-old male is admitted to the ICU with community acquired pneumonia and sepsis. There is a concern for the possible development of stress gastritis. Which of the following is an important risk factor in the development of stress gastritis? A Platelets < 150,000 per microliter B INR < 1.5 C Patient remaining NPO for > 24 hours D Hematocrit < 35% E Respiratory failure requiring mechanical ventilation > 48 hours

The Correct Answer is: E Two of the most important risk factors in the development of stress gastritis are coagulopathy (platelets < 50,000/mcl or INR > 1.5) and respiratory failure, requiring mechanical ventilation for > 48 hours. Hematocrit and amount of time that a patient receives nothing by mouth are not important risk factors.

36-year-old man presents to the emergency department with a tight bandage around his chest to help reduce pain from a chest wall injury on his right side that occurred during mixed martial arts sparring. Physical exam reveals dullness to percussion, dry crackles and diminished breath sounds over the right lower lobe. Chest x-ray shows elevation of the right hemi-diaphragm. What is the most likely diagnosis? A Atelectasis B Bronchiectasis C Pleural Effusion D Pneumothorax E Pulmonary edema

The correct answer is (A). The patient's injury places him at risk of atelectasis, pneumothorax, or other traumatic injuries. The physical exam and chest x-ray findings are classic for atelectasis (A). Pleural effusion (C) would present with fluid in the costophrenic angle on chest x-ray. Pneumothroax (D) would typically present with findings in the upper lung fields including hyperresonance to percussion. Pulmonary edema (E) would present with increased vascular markings and evidence of fluid within the alveolar space on chest x-ray.

24-year-old female HIV-positive patient, who is not currently on medication, presents to the emergency department with acute dyspnea, tachycardia, fever, nonproductive cough, and a room air oxygen saturation of 92%. She admits feeling poorly for the past five days. A physical exam reveals bilateral basilar crackles. An x-ray reveals the image shown. What is the most likely causative organism of this clinical picture? A Chlamydia psittaci B Histoplasmosis C Klebsiella pneumoniae D Pneumocystis jiroveci E Streptoccocus pneumoniae

The correct answer is (D). Pneumocystis is an opportunistic fungal infection of immunocompromised patients, most frequently seen in patients with untreated HIV/AIDS. Patients often present with fever, dyspnea, a nonproductive cough, decreased arterial oxygen pressure, and tachycardia. Lung auscultation may reveal adventitious sounds, but may also be without abnormality. Chest x-ray results classically include bilateral diffuse infiltrates with perihilar involvement. Although the other etiologies are associated with pneumonia, and may be seen in immunocompromised patients, pneumocystis pneumonia occurs in up to 80% of untreated HIV patients and is a leading cause of death

A patient with severe hemophilia A is brought into the emergency department following an automobile accident. Concern is for hemorrhage. What is the best intervention to raise factor VIII levels to hemostatic levels? A Fresh frozen plasma B Cryoprecipitate C Recombinate D DDAVP (desmopressin) E EACA (aminocaproic acid)

The correct answer is (E). While fresh frozen plasma and cryoprecipitate have been used in the past for hemophilia A hemostatis, volumes may be too large or unable to reach levels to achieve hemostasis in severe hemophilic A patients. Recombinate is a commercial lyophilized factor VIII concentrate that can case factor VIII levels to reach hemostasis in smaller volumes, and do not have the disadvantages of plasma or cryopreciptate. DDAVP is not used in severe hemophilics. EACA is used to enhance hemostasis, but is not able to achieve initial hemostasis seen with recombinate therapy.


Conjuntos de estudio relacionados

Chapter 10 respiratory system ; BIO 135: Basic Anatomy/Physiology w/Lab (4226_25ZA); exam #3 review

View Set

Chapter 54: Management of Patients with Kidney Disorders.

View Set